Reg 6 Flashcards

1
Q
Green was adjudicated incompetent by a court having proper jurisdiction. Which of the following statements is correct regarding contracts subsequently entered into by Green?
	a.	
All contracts are enforceable.
	b.	
All contracts are voidable.
	c.	
All contracts are valid.
	d.	
All contracts are void.
A

Choice “d” is correct. Contracts entered into by one who has been adjudicated mentally incompetent are void rather than voidable. Thus, choice “b” is incorrect. Note that choices “c” (all contracts are valid) and “a” (all contracts are enforceable) are the same answer. Since both cannot be correct, both must be wrong.

How well did you know this?
1
Not at all
2
3
4
5
Perfectly
2
Q
Which of the following actions if taken by one party to a contract generally will discharge the performance required of the other party to the contract?
	a.	
Tender.
	b.	
Material breach of the contract.
	c.	
Assignment of rights.
	d.	
Delay in performance.
A

Choice “b” is correct. A material breach generally will discharge the nonbreaching party.
Choice “d” is not as good an answer as “b”. While a delay in performance could cause a discharge, it will do so only in a UCC Sales contract, a contract stating that time is of the essence, or if the delay otherwise materially breaches the contract.
Choice “a” is incorrect. Generally if a party tenders performance, the other party will also have to perform. Tender of performance does not discharge the other party.
Choice “c” is incorrect. Generally, contracts are assignable, and an assignment of rights will not discharge the other party from performing.

How well did you know this?
1
Not at all
2
3
4
5
Perfectly
3
Q

Which of the following facts must be proven for a plaintiff to prevail in a common law negligent misrepresentation action?
a.
The misrepresentations were in writing.
b.
The misrepresentations concerned opinion.
c.
The defendant made the misrepresentations with a reckless disregard for the truth.
d.
The plaintiff justifiably relied on the misrepresentations.

A

Choice “d” is correct. To make out an action for negligent misrepresentation, the plaintiff must show both actual and justifiable reliance on the misrepresentation.
Choice “c” is incorrect. In negligent misrepresentation, the misrepresentation can arise out of conduct that is negligent (i.e., simple carelessness); reckless disregard for truth is considered the equivalent of fraud and is a much higher standard of misconduct.
Choice “a” is incorrect. Misrepresentations need not be in writing to give rise to a cause of action.
Choice “b” is incorrect. The misrepresentation must be of a material fact; misrepresentation of an opinion generally will not support a cause of action for negligent misrepresentation.

How well did you know this?
1
Not at all
2
3
4
5
Perfectly
4
Q

A building subcontractor submitted a bid for construction of a portion of a high-rise office building. The bid contained material computational errors. The general contractor accepted the bid with knowledge of the errors. Which of the following statements best represents the subcontractor’s liability?
a.
Not liable because the errors were a result of gross negligence.
b.
Liable because the errors were material.
c.
Liable because the errors were unilateral.
d.
Not liable because the contractor knew of the errors.

A

Choice “d” is correct. Unilateral mistake is a defense to a contract if the nonmistaken party knew or should have known of the mistake. Here, the contractor knew of the error.
Choice “a” is incorrect. Whether the mistake was due to ordinary negligence or gross negligence is irrelevant in determining whether the mistake will constitute a contract defense.
Choice “c” is incorrect. Unilateral mistake is a defense to a contract if the nonmistaken party knew or should have known of the mistake.
Choice “b” is incorrect. While mistake is grounds for a defense only if the mistake is material, materiality is not all that is necessary. When the mistake is unilateral, as it is here, the nonmistaken party must also have known of the mistake (or had cause to know of the mistake).

How well did you know this?
1
Not at all
2
3
4
5
Perfectly
5
Q

Grove is seeking to avoid performing a promise to pay Brook $1,500. Grove is relying on lack of consideration on Brook’s part. Grove will prevail if he can establish that:
a.
Brook’s only claim of consideration was the relinquishment of a legal right.
b.
The consideration to be performed by Brook will be performed by a third party.
c.
Prior to Grove’s promise, Brook had already performed the requested act.
d.
Brook’s asserted consideration is only worth $400.

A

Choice “c” is correct. A contract generally must be supported by valid consideration. Valid consideration will be present if there is a bargained for exchange of something of legal value. If the act promised has already been performed, the bargain element fails. Thus, it is said that past consideration is no consideration.
Choice “a” is incorrect. A contract generally must be supported by valid consideration. Relinquishment of a legal right constitutes something of legal value. Thus, this is not a good defense for Grove.
Choice “d” is incorrect. As long as the consideration is not a sham, the courts will not inquire into the adequacy of the consideration exchanged. $400 is not sham consideration; thus, the large disparity in value of the consideration exchanged here is not a defense.
Choice “b” is incorrect. The benefits of a contract need not flow to the parties to constitute consideration, the mere giving of a benefit or receipt of a detriment is sufficient.

How well did you know this?
1
Not at all
2
3
4
5
Perfectly
6
Q

One of the criteria for a valid assignment of a sales contract to a third party is that the assignment must:
a.
Be in writing and signed by the assignor.
b.
Be supported by adequate consideration from the assignee.
c.
Not materially increase the other party’s risk or duty.
d.
Not be revocable by the assignor.

A

Choice “c” is correct. Generally, all contracts are assignable unless the assignment would result in a change in the obligor’s risk.
Choice “b” is incorrect. An assignment need not be supported by consideration; assignments may be gratuitous.
Choice “a” is incorrect. There is no requirement that an assignment of a contract for the sale of goods be in writing.
Choice “d” is incorrect. An assignment may be revocable or irrevocable.

How well did you know this?
1
Not at all
2
3
4
5
Perfectly
7
Q
If a person is induced to enter into a contract by another person because of the close relationship between the parties, the contract may be voidable under which of the following defenses?
	a.	
Fraud in the inducement.
	b.	
Undue influence.
	c.	
Duress.
	d.	
Unconscionability.
A

Choice “b” is correct. Undue influence is when a person in a position of trust or confidence takes unfair advantage of the relationship such that the other party’s free will to contract is overcome.

How well did you know this?
1
Not at all
2
3
4
5
Perfectly
8
Q

Under a personal services contract, which of the following circumstances will cause the discharge of a party’s duties?
a.
Illegality of the services to be performed.
b.
Death of the party who is to receive the services.
c.
Cost of performing the services has doubled.
d.
Bankruptcy of the party who is to receive the services.

A

Choice “a” is correct. Illegality of the services to be performed always results in a discharge of duties. This assumes that the services were legal at the time the contract was formed. If the services had been illegal at the time of attempted formation, there would be no contract.
Choice “b” is incorrect. The death of the party who is to receive the services does not usually result in discharge of duties under a personal services contract. However, death might make performance impossible therefore causing a discharge (e.g., if doctor contracts to perform a bypass operation on patient and patient dies before the operation can be performed, doctor is discharged from performing).
Choice “c” is incorrect. A party can be discharged from a contract for impossibility or commercial impracticability, but a mere increase in costs does not make a performance impossible or impracticable.
Choice “d” is incorrect. Mere bankruptcy of the party to receive the services will not result in a discharge, although discharge is possible if the bankruptcy constitutes anticipatory repudiation, such as when it makes it very unlikely that the person receiving the services will be able to pay.

How well did you know this?
1
Not at all
2
3
4
5
Perfectly
9
Q
Ordinarily, in an action for breach of a construction contract, the statute of limitations time period would be computed from the date the:
	a.	
Contract is breached.
	b.	
Contract is signed.
	c.	
Construction is begun.
	d.	
Contract is negotiated
A

Choice “a” is correct. The statute of limitations for breach of contract usually begins to run on the occurrence of the breach

How well did you know this?
1
Not at all
2
3
4
5
Perfectly
10
Q

Egan, a minor, contracted with Baker to purchase Baker’s used computer for $400. The computer was purchased for Egan’s personal use. The agreement provided that Egan would pay $200 down on delivery and $200 thirty days later. Egan took delivery and paid the $200 down payment. Twenty days later, the computer was damaged seriously as a result of Egan’s negligence. Five days after the damage occurred and one day after Egan reached the age of majority, Egan attempted to disaffirm the contract with Baker. Egan will:
a.
Be able to disaffirm despite the fact that Egan was not a minor at the time of disaffirmance.
b.
Not be able to disaffirm because Egan had failed to pay the balance of the purchase price.
c.
Be able to disaffirm only if Egan does so in writing.
d.
Not be able to disaffirm because the computer was damaged as a result of Egan’s negligence.

A

Choice “a” is correct. A minor has a reasonable time after reaching the age of majority to disaffirm contracts. One day after reaching majority is within a reasonable time, and so Egan could disaffirm.
Choice “c” is incorrect. There is no requirement that a minor who wishes to disaffirm must do so in writing.
Choice “b” is incorrect. A minor may disaffirm a partially executed contract. The minor will only have a right to get back what he has paid.
Choice “d” is incorrect. A minor may disaffirm even if the subject matter of the contract has been destroyed; the minor’s only duty is to return whatever is left.

How well did you know this?
1
Not at all
2
3
4
5
Perfectly
11
Q

To prevail on the defense of fraud in the inducement, a victim must prove that the:
a.
Defrauder was an expert with regard to the misrepresentations.
b.
Defrauder was in a fiduciary relationship with the victim.
c.
Misrepresentations were in writing.
d.
Defrauder made the misrepresentations with knowledge of their falsity and with an intention to deceive.

A

Choice “d” is correct. The common law defense of fraud requires a showing of intent to deceive. Fraud in the inducement (as opposed to fraud in the execution) merely means that the victim was deceived as to the reason for the transaction.
Choice “a” is incorrect. A person need not be an expert concerning the subject matter of the deceit to be liable for misrepresentation. The person need only knowingly lie.
Choices “c” and “b” are incorrect. Fraud in the inducement means that the victim was deceived as to the reasons for entering into the fraudulent transaction; there is no requirement that the misrepresentation have been made in writing, so “c” is incorrect. Similarly, there is no requirement of a fiduciary relationship, so “b” is incorrect.

How well did you know this?
1
Not at all
2
3
4
5
Perfectly
12
Q
Which of the following offers of proof are inadmissible under the parol evidence rule when a written contract is intended as the complete agreement of the parties:
I.
Proof of the existence of a subsequent oral modification of the contract.
II.
Proof of the existence of a prior oral agreement that contradicts the written contract.
	a.	
Neither I nor II.
	b.	
II only.
	c.	
I only.
	d.	
Both I and II.
A

Choice “b” is correct. The parol evidence rule prohibits evidence of prior oral or written agreements that seek to contradict the terms of a fully integrated contract (i.e., one intended as the complete agreement). Thus, II is prohibited. However, the parol evidence rule does not prohibit introduction of subsequent agreements; thus, I is not prohibited.

How well did you know this?
1
Not at all
2
3
4
5
Perfectly
13
Q

Ames Construction Co. contracted to build a warehouse for White Corp. The construction specifications required Ames to use Ace lighting fixtures. Inadvertently, Ames installed Perfection lighting fixtures, which are of slightly lesser quality than Ace fixtures, but in all other respects meet White’s needs. Which of the following statements is correct?
a.
White’s recovery will be limited to monetary damages because Ames’ breach of the construction contract was not material.
b.
Ames did not breach the construction contract because the Perfection fixtures were substantially as good as the Ace fixtures.
c.
White will not be able to recover any damages from Ames because the breach was inadvertent.
d.
Ames must install Ace fixtures or White will not be obligated to accept the warehouse.

A

Choice “a” is correct. Contracts governed by the common law, especially construction contracts, do not allow rescission for minor breaches, but limit the nonbreaching party to recovery of damages.
Choice “c” is incorrect. Contract law generally does not differentiate between intentional and inadvertent breaches; damages are recoverable for both.
Choice “b” is incorrect. First, the facts say that Perfection fixtures were of a lesser quality than Ace fixtures, and even if this were not true, there still would be a breach; the contract called for Ace fixtures and so only the use of Ace fixtures would constitute full compliance with the contract.
Choice “d” is incorrect. Contracts governed by the common law, especially construction contracts follow the doctrine of substantial performance. A party who receives substantially all of the benefit of the bargain is bound to the contract and can seek only damages for any minor breaches.

How well did you know this?
1
Not at all
2
3
4
5
Perfectly
14
Q

All of the following are effective methods of ratifying a contract entered into by a minor, except:
a.
Failing to disaffirm the contract within a reasonable time after reaching the age of majority.
b.
Expressly ratifying the contract after reaching the age of majority.
c.
Ratifying the contract before reaching the age of majority.
d.
Impliedly ratifying the contract after reaching the age of majority.

A

Choice “c” is correct. A minor can disaffirm any contract until a reasonable time after reaching the age of majority. Thus, a “ratification” prior to reaching majority can be revoked and is not effective.
Choice “b” is incorrect. Express ratification after reaching the age of majority is one way to ratify a contract.
Choice “a” is incorrect. Failing to disaffirm a contract within a reasonable time after reaching the age of majority constitutes a ratification.
Choice “d” is incorrect. Impliedly ratifying after reaching the age of majority (e.g., by retaining the benefits of the contract or failing to timely disaffirm) effectively ratifies a minor’s contract.

How well did you know this?
1
Not at all
2
3
4
5
Perfectly
15
Q

Which of the following statements correctly applies to a typical statute of limitations?
a.
The statute prohibits the admission into evidence of proof of oral statements about the meaning of a written contract.
b.
The statute provides that only the party against whom enforcement of a contract is sought must have signed the contract.
c.
The statute limits the right of a party to recover damages for misrepresentation unless the false statements were intentionally made.
d.
The statute requires that a legal action for breach of contract be commenced within a certain period of time after the breach occurs.

A

Choice “d” is correct. A statute of limitations requires that actions to enforce rights under a contract be brought within a certain time after breach has occurred.
Choice “b” is incorrect. The Statute of Frauds, not statute of limitations, is concerned about who has signed a contract.
Choice “c” is incorrect. The right to recover if there was an intentional false statement involves the concept of scienter.
Choice “a” is incorrect. Oral statements offered to prove the meaning of a written contract involves the parol evidence rule.

How well did you know this?
1
Not at all
2
3
4
5
Perfectly
16
Q

On February 1, Burns contracted in writing with Nagel to sell Nagel a used car. The contract provided that Burns was to deliver the car on February 15 and Nagel was to pay the $800 purchase price not later than March 15. On February 21, Burns assigned the contract to Ross for $600. Nagel was not notified of the assignment. Which of the following statements is correct?
a.
Ross will not be subject to any contract defenses Nagel could have raised against Burns.
b.
The assignment to Ross is invalid because Nagel was not notified.
c.
By making the assignment, Burns impliedly warranted Nagel would pay the full purchase price.
d.
By making the assignment, Burns impliedly warranted a lack of knowledge of any fact impairing the value of the assignment.

A

Choice “d” is correct. Essentially, by the assignment, Burns sold the contract right to collect the $800 from Nagel. In such a case, the assignor warrants that he does not know of anything that would impair the value of the assignment; otherwise, people would attempt to assign contracts whenever they knew of a problem.
Choice “c” is incorrect. There is no implied warranty that the promisor will perform.
Choice “b” is incorrect. The obligor need not be given notice to effectively assign a contract right. However, until the obligor receives notice, no liability is incurred by paying the assignor.
Choice “a” is incorrect. An assignee generally is subject to all of the defenses that the promisor would have against the assignor relating to the contract (e.g., that the car was stolen).

How well did you know this?
1
Not at all
2
3
4
5
Perfectly
17
Q

Master Mfg., Inc. contracted with Accur Computer Repair Corp. to maintain Master’s computer system. Master’s manufacturing process depends on its computer system operating properly at all times. A liquidated damages clause in the contract provided that Accur pay $1,000 to Master for each day that Accur was late responding to a service request. On January 12, Accur was notified that Master’s computer system failed. Accur did not respond to Master’s service request until January 15. If Master sues Accur under the liquidated damage provision of the contract, Master will:
a.
Lose, because Accur’s breach was not material.
b.
Win, because under all circumstances liquidated damage provisions are enforceable.
c.
Win, unless the liquidated damage provision is determined to be a penalty.
d.
Lose, because liquidated damage provisions violate public policy.

A

Choice “c” is correct. A liquidated damages clause is enforceable if at the time of contracting it appears that the amount of damages in case of breach would be difficult to assess and the amount is a reasonable approximation of damages and not a penalty.
Choice “b” is incorrect. A liquidated damages clause is not enforceable if it constitutes a penalty, if actual damages would be easy to assess at the time the contract was made, or if the liquidated damages amount is not a reasonable approximation of actual damages.
Choice “a” is incorrect. The agreement of the parties made time of the essence under the contract, since Master could not operate without its computer system and this fact was made clear to Accur. Thus, the delay is a material breach.
Choice “d” is incorrect. Liquidated damages provisions do not violate public policy if at the time of contracting it appears that the amount of damages in case of breach would be difficult to assess and the amount is a reasonable approximation of damages and not a penalty.

How well did you know this?
1
Not at all
2
3
4
5
Perfectly
18
Q
On September 10, Harris, Inc., a new car dealer, placed a newspaper advertisement stating that Harris would sell 10 cars at its showroom for a special discount only on September 12, 13, and 14. On September 12, King called Harris and expressed an interest in buying one of the advertised cars. King was told that five of the cars had been sold and to come to the showroom as soon as possible. On September 13, Harris made a televised announcement that the sale would end at 10:00 PM that night. King went to Harris' showroom on September 14 and demanded the right to buy a car at the special discount. Harris had sold the 10 cars and refused King's demand. King sued Harris for breach of contract. Harris' best defense to King's suit would be that Harris':
	a.	
Television announcement revoked the offer.
	b.	
Offer had not been accepted.
	c.	
Offer was unenforceable.
	d.	
Advertisement was not an offer.
A

Choice “d” is correct. Advertisements are generally not offers, but invitations to negotiate. An advertisement is an offer only if it is a promise to perform a very specific act conditioned upon acceptance. If Harris’ ad had stated that Harris would sell 10 specifically identified cars for a specified price during the sale, Harris’ advertisement would be an offer. Harris’ actual advertisement is too vague to be an offer.
Choice “c” is incorrect. The advertisement here is too vague to be considered an offer since it does not specify which cars or what the special discount is.
Choice “a” is incorrect. The announcement did not revoke an offer because the original ad was too vague to be considered an offer since it does not specify which cars or what the special discount is. Moreover, generally revocations must be made through the same means as the offer. If the newspaper ad were an offer, a television announcement would not be a sufficient way to revoke it.
Choice “b” is incorrect. The advertisement here is too vague to be considered an offer since it does not specify which cars or what the special discount is. Thus, there was no offer to be accepted or not accepted. Moreover, even if the ad were an offer, because all 10 cars had already been sold by the time King came in to accept, any attempted acceptance was too late; the offer had terminated because all 10 cars were sold.

How well did you know this?
1
Not at all
2
3
4
5
Perfectly
19
Q

On April 1, Fine Corp. faxed Moss an offer to purchase Moss’ warehouse for $500,000. The offer stated that it would remain open only until April 4 and that acceptance must be received to be effective. Moss sent an acceptance on April 4 by overnight mail and Fine received it on April 5. Which of the following statements is correct?
a.
A contract was formed when Fine received Moss’ acceptance.
b.
No contract was formed because Fine received Moss’ acceptance after April 4.
c.
No contract was formed because Moss sent the acceptance by an unauthorized method.
d.
A contract was formed when Moss sent the acceptance.

A

Choice “b” is correct. Generally, under the mailbox rule, acceptance is effective when sent. However, an offeror may opt out of the mailbox rule by stating that the acceptance must be received by a certain date to be effective. Fine’s offer here required receipt by April 4. Moss’ acceptance was received after the April 4th deadline.
Choice “c” is incorrect. The offer did not require any particular method of acceptance, only receipt within a specified time. Thus, any reasonable means of acceptance was authorized.
Choice “d” is incorrect. Since the offeror required that the acceptance be received to be effective, the mailbox rule would not be applicable.
Choice “a” is incorrect. The offer stated that an acceptance had to be received by April 4 and the attempted acceptance here was received on April 5. Thus, it was ineffective because the offer had already terminated on the date attempted acceptance was received.

How well did you know this?
1
Not at all
2
3
4
5
Perfectly
20
Q

In which of the following situations does the first promise serve as valid consideration for the second promise?
a.
A police officer’s promise to catch a thief for a victim’s promise to pay a reward.
b.
A debtor’s promise to pay $500 for a creditor’s promise to forgive the balance of a $600 liquidated debt.
c.
A builder’s promise to complete a contract for a purchaser’s promise to extend the time for completion.
d.
A debtor’s promise to pay $500 for a creditor’s promise to forgive the balance of a $600 disputed debt.

A

Choice “d” is correct. Anything having legally recognized value can constitute consideration. If parties legitimately disagree as to the amount owed under their contract, a promise to compromise, such as the parties are doing here, has legal value and constitutes consideration since both parties are giving up the right to litigate the dispute.
Choice “a” is incorrect. A promise to do something that one is already obligated to do has no legal value and is not valid consideration under the preexisting legal duty rule. A police officer has a preexisting legal duty to catch thieves; therefore, this promise cannot serve as consideration.
Choice “c” is incorrect. A promise to do something one is already obligated to do has no value and is not valid consideration under the preexisting legal duty rule. Here, the builder already owed a duty to complete the contract, and so his second promise to do so is not valid consideration.
Choice “b” is incorrect. If parties legitimately disagree as to the amount owed under their contract, a promise to compromise has legal value and constitutes consideration since both parties are giving up the right to litigate the dispute. However, here the amount owed is liquidated, which means that it is not in dispute. A promise to compromise here has no legal value and cannot serve as consideration since there is no legitimate right to litigate a liquidated claim.

How well did you know this?
1
Not at all
2
3
4
5
Perfectly
21
Q

West, an Indiana real estate broker, misrepresented to Zimmer that West was licensed in Kansas under the Kansas statute that regulates real estate brokers and requires all brokers to be licensed. Zimmer signed a contract agreeing to pay West a 5% commission for selling Zimmer’s home in Kansas. West did not sign the contract. West sold Zimmer’s home. If West sued Zimmer for nonpayment of commission, Zimmer would be:
a.
Liable to West for the full commission.
b.
Liable to West only for the value of services rendered.
c.
Not liable to West for any amount because West did not sign the contract.
d.
Not liable to West for any amount because West violated the Kansas licensing requirements.

A

Choice “d” is correct. The buyer Zimmer is not liable to West, the unlicensed real estate broker, because West violated Kansas licensing requirements. Failure to have a license required to protect the public (as opposed to merely raise revenue) renders the contract void. Real estate broker licenses are required to protect the public.
Choices “b” and “a” are incorrect, because this licensing law was not designed to raise revenue and, therefore, the contract is void and a party cannot be held liable on a void contract.
Choice “c” is incorrect. Under the Statute of Frauds, certain contracts must be signed by the party sought to be bound to be enforceable. The contract here-to provide brokerage services-probably is not within the Statute since it is for services and can be performed within a year. But even if the contract were within the Statute, it still would be enforceable against Zimmer since Zimmer signed it; it is irrelevant that West did not sign.

How well did you know this?
1
Not at all
2
3
4
5
Perfectly
22
Q
Carson agreed orally to repair Ives' rare book for $450. Before the work was started, Ives asked Carson to perform additional repairs to the book and agreed to increase the contract price to $650. After Carson completed the work, Ives refused to pay and Carson sued. Ives' defense was based on the Statute of Frauds. What total amount will Carson recover?
	a.	
$650
	b.	
$0
	c.	
$450
	d.	
$200
A

Choice “a” is correct. The contract here is for services (repair of a book) and so is governed by the common law. The common law requires modifications to be supported by consideration on both sides. There is consideration on both sides here since Carson agreed to perform additional repairs and Ives agreed to pay more. The Statute of Frauds is not a problem here since the contract is for services and can be performed within a year (the examiners were trying to trick you by taking the contract over the $500 threshold, but that threshold applies only to contracts for the sale of goods). Thus, the oral contract is enforceable as modified.

How well did you know this?
1
Not at all
2
3
4
5
Perfectly
23
Q
In an action for breach of contract, the statute of limitations time period would be computed from the date of the:
	a.	
Signing of the contract.
	b.	
Commencement of the action.
	c.	
Breach of the contract.
	d.	
Negotiation of the contract.
A

Choice “c” is correct. In an action for breach of contract, the statute of limitations time is computed from the date of the breach. Otherwise, in a long-term contract, the statute could run before the breach occurs. Therefore, “a”, “d”, and “b” are incorrect.

How well did you know this?
1
Not at all
2
3
4
5
Perfectly
24
Q
Which of the following, if intentionally misstated by a seller to a buyer, would be considered a fraudulent inducement to make a contract?
	a.	
Prediction.
	b.	
Appraised value.
	c.	
Nonexpert opinion.
	d.	
Immaterial fact.
A

Choice “b” is correct. Fraud requires misrepresentation of a material fact. Ordinarily, opinions or statements of value do not constitute misrepresentations of material facts unless made by experts. Appraised value would constitute a misrepresentation of a material fact because it was made by an expert.
Choice “c” is incorrect. Fraud requires misrepresentation of a material fact. Opinions are not facts.
Choice “a” is incorrect. Predictions are not material facts but rather are mere guesses as to what the future may hold. Therefore, they cannot be the basis for fraud.
Choice “d” is incorrect. Immaterial facts cannot be the basis for a fraud defense since they are not very relevant to the contract decision-making process.

How well did you know this?
1
Not at all
2
3
4
5
Perfectly
25
Q
If a buyer accepts an offer containing an immaterial unilateral mistake, the resulting contract will be:
	a.	
Void as a matter of law.
	b.	
Void at the election of the buyer.
	c.	
Valid as to both parties.
	d.	
Voidable at the election of the seller.
A

Choice “c” is correct. An immaterial unilateral mistake is not a defense to a contract. A material unilateral mistake can be a defense if the nonmistaken party either knew or should have known of the mistake.
Choices “a” and “b” are incorrect. A contract based on a mistake is voidable at the option of the adversely affected party.
Choice “d” is incorrect. A contract based on a material unilateral mistake is voidable at the option of the adversely affected party. Although the mistake here apparently affects the seller, the facts state that it was immaterial and so is not a ground for rescission.

How well did you know this?
1
Not at all
2
3
4
5
Perfectly
26
Q

Under the parol evidence rule, oral evidence will be excluded if it relates to:
a.
Failure of a condition precedent.
b.
A contemporaneous oral agreement relating to a term in the contract.
c.
Lack of contractual capacity.
d.
A modification made several days after the contract was executed.

A

Choice “b” is correct. The parol evidence rule generally bars evidence of prior or contemporaneous oral statements offered to vary the terms of a fully integrated written contract. Oral evidence is permissible when the contract is incomplete, ambiguous, invalid, or subject to a condition precedent, or when modification is made after the original contract is written. A contemporaneous oral agreement will be excluded.
Choice “a” is incorrect. Oral evidence is admissible to prove the failure of a condition precedent since this does not vary the terms of the contract but rather is a collateral issue. The parol evidence rule bars only prior or contemporaneous oral statements that seek to vary the contract’s terms.
Choice “c” is incorrect. The parol evidence rule bars only prior and contemporaneous oral statements that seek to vary the terms of a contract. The lack of contractual capacity is not relevant to the parol evidence rule.
Choice “d” is incorrect. The parol evidence rule bars evidence of prior or contemporaneous statements that contradict a written contract. It does not exclude evidence of subsequent statements.

How well did you know this?
1
Not at all
2
3
4
5
Perfectly
27
Q
To cancel a contract and to restore the parties to their original positions before the contract, the parties should execute a:
	a.	
Rescission.
	b.	
Revocation.
	c.	
Release.
	d.	
Novation.
A

Choice “a” is correct. A rescission “undoes” a contract and restores the parties to the positions they would have been in if no contract were made.
Choice “d” is incorrect. In a novation, the original parties enter into a new contract that releases at least one of the original parties and substitutes at least one new party. All involved parties must agree.
Choice “c” is incorrect, since a release simply discharges a party. It does not restore the party to their original position.
Choice “b” is incorrect. A revocation refers to the withdrawal of an offer. A contract may not be revoked.

How well did you know this?
1
Not at all
2
3
4
5
Perfectly
28
Q

Kaye contracted to sell Hodges a building for $310,000. The contract required Hodges to pay the entire amount at closing. Kaye refused to close the sale of the building. Hodges sued Kaye. To what relief is Hodges entitled?
a.
Compensatory damages or specific performance.
b.
Consequential damages or punitive damages.
c.
Specific performance and compensatory damages.
d.
Punitive damages and compensatory damages.

A

Choice “a” is correct. When a contract for the sale of real property is breached, the nonbreaching party can either recover compensatory damages (damages that compensate for the breach) or obtain specific performance (forced performance).
Choice “d” is incorrect. In contract cases, punitive damages are generally not awarded.
Choice “c” is incorrect. Damages are designed to compensate for nonperformance while specific performance forces performance. These are alternative remedies. A party cannot obtain specific performance and recover compensatory damages.
Choice “b” is incorrect. In contract cases, punitive damages are generally not awarded. Moreover, a party generally is not limited to consequential damages (the collateral damages that result from a breach); the actual damages that result from the breach are also recoverable (compensatory damages).

How well did you know this?
1
Not at all
2
3
4
5
Perfectly
29
Q
In determining whether the consideration requirement to form a contract has been satisfied, the consideration exchanged by the parties to the contract must be:
	a.	
Legally sufficient.
	b.	
Of approximately equal value.
	c.	
Fair and reasonable under the circumstances.
	d.	
Exchanged simultaneously by the parties.
A

Choice “a” is correct. To be effective, consideration must be legally sufficient, which means something that the law recognizes as consideration.
Choice “b” is incorrect. The courts do not require contracts to be “fair,” giving each party equal benefit.
Choice “d” is incorrect. There is no requirement that consideration be exchanged simultaneously. For example, in a unilateral contract, the promise is given first and the performance, which is the consideration on the other side, comes later.
Choice “c” is incorrect. The courts will not inquire into the fairness of a bargain.

How well did you know this?
1
Not at all
2
3
4
5
Perfectly
30
Q

On June 15, Peters orally offered to sell a used lawn mower to Mason for $125. Peters specified that Mason had until June 20 to accept the offer. On June 16, Peters received an offer to purchase the lawn mower for $150 from Bronson, Mason’s neighbor. Peters accepted Bronson’s offer. On June 17, Mason saw Bronson using the lawn mower and was told the mower had been sold to Bronson. Mason immediately wrote to Peters to accept the June 15 offer. Which of the following statements is correct?
a.
Mason’s acceptance would be effective when received by Peters.
b.
Mason’s acceptance would be effective when mailed.
c.
Peters’ offer had been revoked and Mason’s acceptance was ineffective.
d.
Peters was obligated to keep the June 15 offer open until June 20.

A

Choice “c” is correct. Peters’ offer was already revoked. To be effective, an acceptance must be received before the offer is terminated. An offer may be terminated by the offeror at any time unless the offeree gave consideration to keep the offer open. Mason gave no consideration to keep the offer here open; thus Peters could revoke. An offer is considered to be revoked if the offeree obtains information from a reliable source that the subject matter of the offer has been sold. Here, Mason received the information of the sale before he attempted to accept. Therefore, his attempted acceptance came too late and was ineffective.

How well did you know this?
1
Not at all
2
3
4
5
Perfectly
31
Q

Rail, who was 16 years old, purchased an $800 computer from Elco Electronics. Rail and Elco are located in a state where the age of majority is 18. On several occasions Rail returned the computer to Elco for repairs. Rail was very unhappy with the computer. Two days after reaching the age of 18, Rail was still frustrated with the computer’s reliability, and returned it to Elco, demanding an $800 refund. Elco refused, claiming that Rail no longer had a right to disaffirm the contract. Elco’s refusal is:
a.
Correct, because Rail could have transferred good title to a good faith purchaser for value.
b.
Incorrect, because Rail could disaffirm the contract at any time.
c.
Incorrect, because Rail disaffirmed the contract within a reasonable period of time after reaching the age of 18.
d.
Correct, because Rail’s multiple requests for service acted as a ratification of the contract.

A

Choice “c” is correct. A minor may disaffirm a contract within a reasonable time after reaching majority, and two days is certainly a reasonable time.
Choice “d” is incorrect. A minor cannot ratify a contract until reaching majority. Thus, Rail’s requests for repairs before reaching majority are irrelevant.
Choice “a” is incorrect. A minor has a right to disaffirm contracts despite the minor’s power to transfer good title to a good faith purchaser.
Choice “b” is incorrect. A minor must disaffirm a contract within a reasonable time after reaching majority.

How well did you know this?
1
Not at all
2
3
4
5
Perfectly
32
Q

Which of the following statements is true with regard to the Statute of Frauds?
a.
The written contract must be signed by all parties.
b.
All contracts involving consideration of more than $500 must be in writing.
c.
The Statute of Frauds applies to contracts that can be fully performed within one year from the date they are made.
d.
The contract terms may be stated in more than one document.

A

Choice “d” is correct. There is no requirement that the terms of a written contract be contained in a single writing.
Choice “b” is incorrect. The $500 rule applies only to contracts for the sale of goods under the UCC. (Note: Be careful when selecting answers that contain absolute terms such as all, always, only, etc.)
Choice “a” is incorrect. The Statute of Frauds does not require the contract to be in writing—only that some writing reflects the material terms of the contract and that that writing is signed by the party sought to be held liable, not necessarily both parties. (Again, be careful when selecting answers that contain absolute terms such as all, always, only, etc.)
Choice “c” is incorrect. The Statute of Frauds does not apply to contracts that can be fully performed within one year.

How well did you know this?
1
Not at all
2
3
4
5
Perfectly
33
Q

Maco, Inc. and Kent contracted for Kent to provide Maco certain consulting services at an hourly rate of $20. Kent’s normal hourly rate was $90 per hour, the fair market value of the services. Kent agreed to the $20 rate because Kent was having serious financial problems. At the time the agreement was negotiated, Maco was aware of Kent’s financial condition and refused to pay more than $20 per hour for Kent’s services. Kent has now sued to rescind the contract with Maco, claiming duress by Maco during the negotiations. Under the circumstances, Kent will:
a.
Lose, because Maco cannot prove that Kent, at the time, had no other offers to provide consulting services.
b.
Win, because Maco refused to pay the fair market value of Kent’s services.
c.
Lose, because Maco’s actions did not constitute duress.
d.
Win, because Maco was aware of Kent’s serious financial problems.

A

Choice “c” is correct. Duress occurs when a person overcomes the will of another through wrongful force or threats of imminent force. Economic duress generally is not recognized as a defense to contract, and even where it is, it is usually required that the party taking advantage of the other party’s poor financial condition must have caused the poor condition.
Choice “b” is incorrect. The parties to a contract are free to drive the best bargain they can. It is not a defense that the price agreed to was unfair.
Choice “d” is incorrect. A party is free to drive the best bargain he can in contracting. The fact that the other party is in a poor financial condition does not change this rule.
Choice “a” is incorrect. Kent contracted to provide services for $20 per hour and the state of his other offers for work is unrelated to the contract here.

How well did you know this?
1
Not at all
2
3
4
5
Perfectly
34
Q

Miller negotiated the sale of Miller’s liquor store to Jackson. Jackson asked to see the prior year’s financial statements. Using the store’s checkbook, Miller prepared a balance sheet and profit and loss statement as well as he could. Miller told Jackson to have an accountant examine Miller’s records because Miller was not an accountant. Jackson failed to do so and purchased the store in reliance on Miller’s financial statements. Jackson later learned that the financial statements included several errors that resulted in a material overstatement of assets and net income. Miller was not aware that the errors existed. Jackson sued Miller, claiming Miller misrepresented the store’s financial condition and that Jackson relied on the financial statements in making the decision to acquire the store. Which of the following statements is correct?
a.
Jackson would be entitled to rescind the purchase even if the errors in the financial statements were not material.
b.
Jackson will not prevail because Jackson’s reliance on the financial statements was not reasonable.
c.
Money damages is the only remedy available to Jackson if, in fact, Miller has committed a misrepresentation.
d.
Jackson will prevail if the errors in the financial statements were material.

A

Choice “b” is correct. Since Miller did not intentionally misrepresent the store’s financial condition (i.e., the misrepresentation was unintentional), the misrepresentation is a defense only if reliance on it was reasonable. It was not reasonable to rely here since Miller told Jackson that Miller did not know what he was doing and that Jackson should hire an accountant.
Choice “d” is incorrect. Even if the misrepresentations were material, because Miller’s misrepresentation was innocent, it is actionable only if Jackson reasonably relied on the misrepresentation. Jackson did not reasonably rely on the misrepresentation, because Miller had told him that Miller was not an accountant and that Jackson should have Miller’s work checked.
Choice “c” is incorrect. For misrepresentation, the injured party has a choice of voiding the contract (and getting the purchase price back) or damages.
Choice “a” is incorrect, since Jackson would have to prove that the errors were “material” in order to rescind the agreement.

How well did you know this?
1
Not at all
2
3
4
5
Perfectly
35
Q

Ferco, Inc. claims to be a creditor beneficiary of a contract between Bell and Allied Industries, Inc. Allied is indebted to Ferco. The contract between Bell and Allied provides that Bell is to purchase certain goods from Allied and pay the purchase price directly to Ferco until Allied’s obligation is satisfied. Without justification, Bell failed to pay Ferco and Ferco sued Bell. Ferco will:
a.
Prevail, provided Ferco was aware of the contract between Bell and Allied at the time the contract was entered into.
b.
Prevail, because Ferco was an intended beneficiary of the contract between Allied and Bell.
c.
Not prevail, because Ferco did not give any consideration to Bell.
d.
Not prevail, because Ferco lacked privity of contract with either Bell or Allied.

A

Choice “b” is correct. Intended beneficiaries (persons who were intended to be benefited by a contract other than the bargaining parties) can enforce the contract as soon as their rights vest. Here, Allied clearly intended Ferco to receive benefits of the contract and Ferco’s rights vested upon filing suit for Bell’s breach, if not before.
Choice “d” is incorrect. Intended beneficiaries are in privity (one of the parties to a contract) since they are actually named in the contract.
Choice “c” is incorrect. There is no requirement that an intended beneficiary give consideration; it is sufficient that consideration flows between the other parties.
Choice “a” is incorrect. An intended beneficiary need not know of the contract when it is made; his or her rights may vest in the contract later by relying on it, assenting to it, or bringing suit on it.

How well did you know this?
1
Not at all
2
3
4
5
Perfectly
36
Q

Rogers and Lennon entered into a written computer consulting agreement that required Lennon to provide certain weekly reports to Rogers. The agreement also stated that Lennon would provide the computer equipment necessary to perform the services, and that Rogers’ computer would not be used. As the parties were executing the agreement, they orally agreed that Lennon could use Rogers’ computer. After executing the agreement, Rogers and Lennon orally agreed that Lennon would report on a monthly, rather than weekly, basis. The parties now disagree on Lennon’s right to use Rogers’ computer and how often Lennon must report to Rogers. In the event of a lawsuit between the parties, the parol evidence rule will:
a.
Not apply to any of the parties’ agreements because the consulting agreement did not have to be in writing.
b.
Not prevent Lennon from proving the parties’ oral agreement that Lennon could use Rogers’ computer.
c.
Not prevent the admission into evidence of testimony regarding Lennon’s right to report on a monthly basis.
d.
Not apply to the parties’ agreement to allow Lennon to use Rogers’ computer because it was contemporaneous with the written agreement.

A

Choice “c” is correct. The parol evidence rule prohibits introduction of prior or contemporaneous oral statements that seek to vary the terms of a written contract. It does not prohibit introduction of subsequent oral statements that modify the contract.
Choice “a” is incorrect. The parol evidence rule applies whenever there is a written contract, not only to contracts that must be in writing because of the Statute of Frauds.
Choice “b” is incorrect. The parol evidence rule prohibits introduction of prior or contemporaneous oral statements that seek to vary the terms of a written contract. The agreement that Lennon could use Rogers’ computer is different from what the contract provides and was made contemporaneously with the written contract. Thus, the parol evidence rule will bar its introduction.
Choice “d” is incorrect. The parol evidence rule bars contemporaneous oral statements that seek to vary the terms of a written contract.

How well did you know this?
1
Not at all
2
3
4
5
Perfectly
37
Q

Wilcox Co. contracted with Ace Painters, Inc. for Ace to paint Wilcox’s warehouse. Ace, without advising Wilcox, assigned the contract to Pure Painting Corp. Pure failed to paint Wilcox’s warehouse in accordance with the contract specifications. The contract between Ace and Wilcox was silent with regard to a party’s right to assign it. Which of the following statements is correct?
a.
Ace remained liable to Wilcox despite the fact that Ace assigned the contract to Pure.
b.
Ace’s duty to paint Wilcox’s warehouse was nondelegable.
c.
Ace would not be liable to Wilcox if Ace had notified Wilcox of the assignment.
d.
Ace’s delegation of the duty to paint Wilcox’s warehouse was a breach of the contract.

A

Choice “a” is correct. In the absence of an agreement to the contrary, the assignment of a contract does not relieve the assignor of his/her obligations under the contract. Here, Ace remains liable to Wilcox even though Ace assigned (delegated) its contractual responsibility to Pure.
Choice “c” is incorrect. Notice alone isn’t enough to remove liability. Ace could avoid liability only by an agreement among all of the parties substituting Pure for Ace and releasing Ace. Such an agreement is known as a novation.
Choice “b” is incorrect. Only duties that involve specialized personal skills (i.e., rely heavily on the personal attributes of the person performing) cannot be delegated, and a contract to paint a warehouse does not rely on specialized skills, as would a contract to paint a portrait.
Choice “d” is incorrect. The duty to paint a building does not involve highly specialized skills and, therefore, is delegable. Since Ace’s duty to paint Wilcox’s warehouse could be delegated, the act of delegating was not a breach of contract.

How well did you know this?
1
Not at all
2
3
4
5
Perfectly
38
Q
Yost contracted with Egan for Yost to buy certain real property. If the contract is otherwise silent, Yost's rights under the contract are:
	a.	
Assignable only with Egan's consent.
	b.	
Nonassignable because they are personal to Yost.
	c.	
Generally assignable.
	d.	
Nonassignable as a matter of law.
A

Choice “c” is correct. Most contracts are assignable unless they involve personal services or the assignment would vary the risks or burdens of the contract. Normally a real estate contract is assignable.
Choices “a”, “b”, and “d” are incorrect. Generally, unless a contract involves specialized personal services or otherwise relies heavily on the personal attributes of the parties, it is assignable without consent. A contract to purchase real estate does not involve personal services, therefore no consent is needed.

How well did you know this?
1
Not at all
2
3
4
5
Perfectly
39
Q

Kram sent Fargo, a real estate broker, a signed offer to sell a specified parcel of land to Fargo for $250,000. Kram, an engineer, had inherited the land. On the same day that Kram’s letter was received, Fargo telephoned Kram and accepted the offer. Which of the following statements is correct under the common law statute of frauds?
a.
A contract was formed but would be enforceable only against Kram.
b.
A contract was formed and would be enforceable against both Kram and Fargo.
c.
No contract could be formed because Fargo’s acceptance was oral.
d.
No contract could be formed because Kram’s letter was signed only by Kram.

A

Choice “a” is correct. A contract was formed between Kram and Fargo because Kram made a valid offer and Fargo made a valid acceptance of that offer. Under the Statute of Frauds, contracts involving interests in real property are enforceable only if their material terms are set forth in a writing signed by the party sought to be held liable. The original signed offer from Kram would be a sufficient writing to satisfy the Statute of Frauds if Fargo is seeking to enforce the contract. Since Fargo accepted by telephone, there is no writing signed by Fargo. Thus, the contract could be enforced against Kram, but could not be enforced against Fargo due to the lack of a signed writing by Fargo.
Choices “c” and “d” are incorrect because they state that no contract was formed. There was a contract. The Statute of Frauds does not make oral contracts invalid, it only makes them unenforceable.
Choice “b” is incorrect because the contract could not be enforced against Fargo. A real estate contract is within the Statute of Frauds, and a contract within the Statute of Frauds generally is enforceable against a party only if that party signed a written memorandum containing the material terms of the contract.

How well did you know this?
1
Not at all
2
3
4
5
Perfectly
40
Q

On May 25, Fresno sold Bronson, a minor, a used computer. On June 1, Bronson reached the age of majority. On June 10, Fresno wanted to rescind the sale. Fresno offered to return Bronson’s money and demanded that Bronson return the computer. Bronson refused, claiming that a binding contract existed. Bronson’s refusal is:
a.
Justified, because Bronson and Fresno are bound by the contract as of the date Bronson reached the age of majority.
b.
Justified, because Fresno must perform under the contract regardless of Bronson’s minority.
c.
Not justified, because Fresno does not have to perform under the contract if Bronson has a right to disaffirm the contract.
d.
Not justified, because Fresno is not bound by the contract unless Bronson specifically ratifies the contract after reaching the age of majority.

A

Choice “b” is correct. In this problem, Fresno was an adult and Bronson was a minor at the time of contracting. The common law gives minors the right to disaffirm a contract anytime while a minor or within a reasonable time after becoming an adult. Only the minor has the right to disaffirm. The adult may not disaffirm the contract.
Choice “d” is incorrect. Fresno, the adult, is bound by the contract unless Bronson disaffirms. Bronson does not have to ratify.
Choice “c” is incorrect. Fresno, the adult, is bound by the contract unless Bronson disaffirms. Fresno does not have the right to get out of the contract. Only Bronson, the minor, does.
Choice “a” is incorrect. Fresno was bound by the contract on the date it was made. Bronson becomes bound upon ratifying or failing to disaffirm the contract within a reasonable time after reaching the age of majority.

How well did you know this?
1
Not at all
2
3
4
5
Perfectly
41
Q
West, Inc. and Barton entered into a contract. After receiving valuable consideration from Egan, West assigned its rights under the Barton contract to Egan. In which of the following circumstances would West not be liable to Egan?
	a.	
West released Barton.
	b.	
Egan released Barton.
	c.	
Barton paid West.
	d.	
West breached the contract.
A

Choice “b” is correct. In an assignment for consideration, the assignor and obligor are both liable to the assignee. However, if the assignee releases the obligor, it will serve to release the assignor as well. Thus, if Egan (the assignee) releases Barton (the obligor), West (the assignor) will be released as well.
Choice “a” is incorrect. In an assignment for consideration, the assignor and obligor are both liable to the assignee. Thus, if West (the assignor) released Barton (the obligor), West would remain liable to Egan (the assignee) because Egan paid consideration to be assigned West’s rights.
Choice “d” is incorrect. In an assignment for consideration, the assignor owes a contractual duty to the assignee. If the assignor breaches his contract with the obligor so that the obligor is discharged from performing for the assignee, the assignee can hold the assignor liable.
Choice “c” is incorrect. In an assignment for consideration, the assignor and the obligor are both liable to the assignee. Thus, if Barton (the obligor) pays West (the assignor) it does nothing to extinguish the obligation owed to Egan (the assignee). Therefore, it would not constitute a release of West (the assignor).

How well did you know this?
1
Not at all
2
3
4
5
Perfectly
42
Q
Which of the following types of mistake will generally make a contract unenforceable and allow it to be rescinded?
	a.	
A mutual mistake of value.
	b.	
A unilateral mistake of fact.
	c.	
A mutual mistake of fact.
	d.	
A unilateral mistake of value.
A

Choice “c” is correct. A mutual mistake of a material fact will make a contract voidable at the option of the adversely affected party.
Choice “b” is incorrect. Generally, a unilateral mistake of fact does not make the contract voidable. The mistaken party can avoid a contract on the basis of a unilateral mistake only if the other party either knew or should have known of the mistake.
Choice “d” is incorrect. Only mistakes as to material facts can make a contract unenforceable. Value generally is not a fact but, rather, is a matter of opinion.
Choice “a” is incorrect. Only mistakes as to material facts can make a contract unenforceable. Value generally is not a fact but, rather, is a matter of opinion.

How well did you know this?
1
Not at all
2
3
4
5
Perfectly
43
Q

Which of the following contract rights can generally be assigned?
a.
The right of an insured to coverage under a fire insurance policy.
b.
A right whose assignment is prohibited by statute.
c.
The right to receive personal services.
d.
The right to receive a sum of money.

A

Choice “d” is correct. Generally, one can assign rights under a contract with the exception of personal services or when the assignment increases the obligor’s risk. The right to receive money is a very common right that is assignable.
Choice “c” is incorrect. Generally, the right to receive personal services is not assignable.
Choice “a” is incorrect. Generally one cannot assign a right if the assignment will vary the obligor’s risk. Insurance policies are contracts involving the assessment of risk. Therefore, coverage rights under these contracts are not assignable.
Choice “b” is incorrect. If a statute prohibits assignment of a particular right then, obviously, the right is not assignable by operation of law.

How well did you know this?
1
Not at all
2
3
4
5
Perfectly
44
Q

Which of the following promises is supported by legally sufficient consideration and will be enforceable?
a.
A promise to pay the police $250 to catch a thief.
b.
A person’s promise to pay a real estate agent $1,000 in return for the real estate agent’s earlier act of not charging commission for selling the person’s house.
c.
A parent’s promise to pay one child $500 because that child is not as wealthy as the child’s sibling.
d.
A promise to pay a minor $500 to paint a garage.

A

Choice “d” is correct. To constitute consideration, there must be a bargained for exchange of something of value. A detriment to the promisee or a benefit to the promisor constitutes value. A promise to pay a minor $500 to paint a garage constitutes a detriment to the promisee; the promisee is not otherwise bound to pay the minor $500 to paint the garage, and the minor’s painting the garage constitutes valid consideration to support the promise to pay $500.
Choice “b” is incorrect. To constitute consideration, there must be a bargained for exchange of value. A detriment to the promisee or a benefit to the promisor constitutes value. Past acts generally do not constitute valid consideration because the acts were not bargained for. Thus, a promise to pay $1,000 in return for a prior performed act (not charging for services) is not supported by consideration.
Choice “c” is incorrect. To constitute consideration, there must be a bargained for exchange of value. A detriment to the promisee or a benefit to the promisor constitutes value. Here, there is no bargained for exchange. The promise is gratuitous and not supported by consideration.
Choice “a” is incorrect. To constitute consideration, there must be a bargained for exchange of value. A detriment to the promisee or a benefit to the promisor constitutes value. A promise to perform a pre-existing duty does not constitute valid consideration. Because a police officer already owes a crime victim a duty to catch the perpetrator, the police officer’s promise to perform his duty does not constitute valid consideration.

How well did you know this?
1
Not at all
2
3
4
5
Perfectly
45
Q

For which of the following contracts will a court generally grant the remedy of specific performance?
a.
A contract for the sale of fungible goods.
b.
A contract of employment.
c.
A contract for the sale of stock that is traded on a national stock exchange.
d.
A contract for the sale of a patent.

A

Choice “d” is correct. Specific performance is a court order to perform under the terms of a contract. Generally, it is available only in contracts for unique or rare property. A patent, by definition, is unique. Therefore, specific performance would be available to enforce a contract for the sale of a patent.
Choice “b” is incorrect. Specific performance is not available to enforce a contract of employment, at least not in an action by the employer, because it would be tantamount to an order of involuntary servitude, which is prohibited by the 14th Amendment to the United States Constitution.
Choice “a” is incorrect. Specific performance is available only to enforce a contract involving unique or rare property. It is not available to enforce a contract for fungible goods, as such goods are easily replaced.
Choice “c” is incorrect. Specific performance is available only to enforce a contract involving unique or rare property. It is not available to enforce a contract for fungible items. Stock traded on a national securities exchange is fungible.

How well did you know this?
1
Not at all
2
3
4
5
Perfectly
46
Q

Camp orally guaranteed payment of a loan Camp’s cousin Wilcox had obtained from Camp’s friend Main. The loan was to be repaid in 10 monthly payments. After making six payments, Wilcox defaulted on the loan and Main demanded that Camp honor the guaranty. Regarding Camp’s liability to Main, Camp is:
a.
Liable under the oral guaranty because Camp benefitted by maintaining a personal relationship with Main.
b.
Not liable under the oral guaranty because Camp’s guaranty must be in writing to be enforceable.
c.
Liable under the oral guaranty because the loan would be paid within one year.
d.
Not liable under the oral guaranty because of failure of consideration.

A

Explanation
Choice “b” is correct. A promise to pay the debt of another must be evidenced by some type of writing to be enforceable. Camp’s oral promise to pay the debt of Wilcox is not enforceable without some type of writing.
Choices “c” and “a” are incorrect because they state Camp is liable. Camp is not liable due to the lack of a writing.
Choice “d” is incorrect. Consideration is present. Each of the parties promised to do something of legal value. Main promised to loan Wilcox money. Wilcox promised to repay the loan. Camp promised to pay if Wilcox defaulted. Each of the promises serves to make the others enforceable.

How well did you know this?
1
Not at all
2
3
4
5
Perfectly
47
Q

On December 1, Gem orally contracted with Mason for Mason to manage Gem’s restaurant for one year starting the following January 1. They agreed that Gem would pay Mason $40,000 and that Mason would be allowed to continue to work for Gem if “everything worked out.” On June 1, Mason quit to take a better paying job, alleging that the contract violated the statute of frauds. What will be the outcome of a suit by Gem for breach of contract?
a.
Gem will lose because the contract required payment of more than $500.
b.
Gem will win because the contract was for services not goods.
c.
Gem will lose because the contract could not be performed within one year.
d.
Gem will win because the contract was executory.

A

Choice “c” is correct. As a general rule, under the statute of frauds, a contract that cannot be performed within one year from the time of its making is unenforceable absent proof of its material terms in a writing signed by the party being sued. Here, the contract by its terms could not be performed within a year from the time it was made and Gem cannot prove the material terms of the contract through a writing signed by Mason. Therefore, Gem would lose its breach of contract action.

How well did you know this?
1
Not at all
2
3
4
5
Perfectly
48
Q

Card communicated an offer to sell Card’s stereo to Bend for $250. Which of the following statements is correct regarding the effect of the communication of the offer?
a.
Bend should immediately accept or reject the offer to avoid liability to Card.
b.
Bend may not reject the offer for a reasonable period of time.
c.
Card is not obligated to sell the stereo to Bend until Bend accepts the offer.
d.
Card is required to mitigate any loss Card would sustain in the event Bend rejects the offer.

A

Choice “c” is correct. In order to form a contract, there must be at least an offer, an acceptance, and consideration. Card’s communication is an offer. The stereo and the $250 would be the consideration for the contract here. But, Card will not be bound until Bend accepts the offer.
Choice “a” is incorrect. As a general rule, silence cannot constitute an acceptance, and Bend cannot be liable on a contract until it is accepted. If Bend remains silent, no contract is formed and Bend has no liability.
Choice “d” is incorrect. Card owes no contractual duties to Bend until Bend has accepted Card’s offer. Thus, there is no duty to mitigate here.
Choice “b” is incorrect. An offeree may reject an offer at any time.

49
Q
Pierce owed Duke $3,000. Pierce contracted with Lodge to paint Lodge's house and Lodge agreed to pay Duke $3,000 to satisfy Pierce's debt. Pierce painted Lodge's house but Lodge did not pay Duke the $3,000. In a lawsuit by Duke against Pierce and Lodge, who will be liable to Duke?
	a.	
Pierce only.
	b.	
Lodge only.
	c.	
Both Pierce and Lodge.
	d.	
Neither Pierce nor Lodge.
A

Choice “c” is correct. Duke is a creditor of Pierce and Pierce made a contract with Lodge benefiting Duke, making Duke a creditor beneficiary of the Pierce-Lodge contract. If there is a breach of a contract benefiting a third party creditor, the creditor beneficiary may sue the promisor under the third party beneficiary contract or the party who owed the creditor the original debt under the original debt. Thus, none of the other choices are correct.

50
Q

Jen is at a client preparing to begin her audit of their financial statements. Her new laptop computer is not booting up when she presses the start button. She bangs the lid down and vows, loud enough for Paul to hear, that she would sell the computer to anyone for a dollar. Paul reaches into his wallet, places one dollar down on Jen’s desk, and declares to her, “Sold!” Paul’s act:
a.
Is not a valid acceptance, because Jen does not seriously intend to sell the computer.
b.
Is a valid acceptance.
c.
Is not a valid acceptance, because Jen’s offer did not sufficiently identify the offeree.
d.
Is a valid acceptance, because Paul’s statement of “Sold” is universally known to commit a seller of goods to sell those goods to the buyer.

A

Choice “a” is correct. To be sufficient to be an offer, the communication must create a reasonable expectation in the offeree that the offeror intends to make a contract. Given the circumstances (a laptop for $1; not at a sale, etc.) it would be unreasonable to assume that Jen was serious.
Choices “b” and “d” are incorrect. A reasonable person would not assume that Jen’s statement offering to sell the computer was serious. Therefore, there is no offer for Paul to validly accept.
Choice “c” is incorrect. An offer need not name the offeree; it is sufficient if it limits who may accept. As Jen had only one computer, her statement would be understood as limiting the offeree to the first person who gave her one dollar.

51
Q

Jen offers to sell Bob her laptop computer for $500. Bob is not sure he can afford it and asks Jen if she will keep the offer open until noon the next day. Jen tells Bob that she will keep her offer open if he gives her his promise that he won’t buy another computer before that time. Bob gives her his promise not to buy another computer. Jen is:
a.
Obligated to keep the offer open because she gave her word.
b.
Obligated to keep the offer open until noon the next day because there is a valid option contract in place.
c.
Not obligated to keep the offer open because Bob did not give valid consideration to create an option contract.
d.
Not obligated to keep the offer open to Bob because he didn’t buy it right away.

A

Choice “b” is correct. Generally, a promise to keep an offer open for a specified time is unenforceable unless it is made in writing by a merchant (i.e., a merchant’s firm offer) or consideration supports the promise, in which case, an option contract is formed. Here, Bob gave his promise to not purchase another computer in exchange for Jen’s promise to keep the promise open. Bob was not legally obligated to forgo purchasing another computer, and giving up the right to do so is valid consideration. Thus, the parties created an option contract to keep the offer open.

52
Q

Karen tells her friend Judi that she plans to shave her dog’s long hair so short Judi will be able to see the freckles on his skin. Judi feels so bad for the dog that she offers to pay Karen $50 to have a professional dog groomer cut the dog’s hair to a more attractive length and style. Karen immediately leaves for the groomer and has the dog’s hair professionally groomed. Judi’s promise to pay is:
a.
Enforceable because Karen was only saying she was shaving the dog to get Judi to offer her some money.
b.
Enforceable because Karen had a professional groomer cut the dog’s hair.
c.
Unenforceable because Karen should not have considered shaving the dog’s hair in the first place.
d.
Unenforceable even though Karen had a professional groomer cut the dog’s hair in anticipation of receiving $50 from Judi.

A

Choice “b” is correct. Karen and Judi formed a unilateral contract. A unilateral contract is formed when a promise is made in exchange for an act. Here, Judi promised to pay Karen $50 if Karen had a professional groomer style the dog’s hair. Karen accepted the offer by having a professional groomer cut her dog’s hair. Karen’s performance of the requested act not only was an acceptance, but it also was valid consideration to support Judi’s promise. Consideration need not have monetary value to the promisor; a detriment to the promisee is sufficient. Here, Karen did not have to have the dog’s hair cut by a professional groomer. Thus, her doing so was a detriment and constituted consideration sufficient to support Judi’s promise.

53
Q
Maureen is 15 years old. She buys a $2,000 bicycle that she continues to use and keep in repair after reaching the age of majority. Maureen has:
	a.	
Disaffirmed the contract.
	b.	
Ratified the contract.
	c.	
Rejected the contract.
	d.	
Rescinded the contract.
A

Choice “b” is correct. Upon reaching the age of majority, a person can become bound on contracts entered into as a minor through ratification by retaining the benefits of the contract.
Choice “a” is incorrect. A minor may disaffirm a contract (i.e., negate her assent to enter into the contract) any time up to a reasonable time after reaching the age of majority. Maureen did nothing here to indicate that she wanted to negate her assent to enter into the contract.
Choice “d” is incorrect. Rescission is an act of “undoing” a contract. Maureen did nothing here to indicate that she wanted to undo the contract.
Choice “c” is incorrect. An offer can be rejected, in which case no contract is formed. Thus, technically, a contract cannot be rejected.

54
Q

With regard to an agreement for the sale of real estate, the Statute of Frauds:
a.
Does not apply if the value of the real estate is less than $500.
b.
Requires that the entire agreement be in a single writing.
c.
Requires that the purchase price be fair and adequate in relation to the value of the real estate.
d.
Does not require that the agreement be signed by all parties.

A

Choice “d” is correct. The Statute of Frauds does not require that an agreement be signed by all parties, merely the party to be charged. The contract, however, generally is unenforceable against a party who did not sign.
Choice “a” is incorrect. The $500 threshold concerns sale of goods and not real estate.
Choice “b” is incorrect. There is no such rule. The contract could be made up of a series of writings, that when taken together reflect the material terms of the contract.
Choice “c” is incorrect. There is no such rule. The Statute of Frauds is concerned with whether certain contracts are evidenced by the party to be held liable under the contract and not with the fairness and adequacy of the price.

55
Q
Wren purchased a factory from First Federal Realty. Wren paid 20% at the closing and gave a note for the balance secured by a 20-year mortgage. Five years later, Wren found it increasingly difficult to make payments on the note and defaulted. First Federal threatened to accelerate the loan and foreclose if Wren continued in default. First Federal told Wren to make payment or obtain an acceptable third party to assume the obligation. Wren offered the land to Moss, Inc. for $10,000 less than the equity Wren had in the property. This was acceptable to First Federal and at the closing Moss paid the arrearage, assumed the mortgage and note, and had title transferred to its name. First Federal released Wren. The transaction in question is a (an):
	a.	
Novation.
	b.	
Assignment and delegation.
	c.	
Third party beneficiary contract.
	d.	
Purchase of land subject to a mortgage
A

Choice “a” is correct. In a novation, a new contract substitutes a new party for an old party in an existing contract and all of the parties agree to release the party who was substituted out. That is what happened here.
Choice “d” is incorrect. A person who takes “subject to” a mortgage does not agree to be personally liable on the mortgage. Here, Moss agreed to be personally liable, making this an assumption of a mortgage rather than taking subject to a mortgage.
Choice “b” is incorrect. In an assignment/delegation situation, the assignor (Wren) continues to be liable on the obligation. Here, Moss was substituted for Wren when Moss assumed the mortgage, so Wren has no rights or duties under the original contract.
Choice “c” is incorrect. The transaction is best described as a novation because there is a substitution of parties.

56
Q

Under a contract governed by the UCC Sales Article, which of the following statements is correct?
a.
The contract will not be enforceable if it fails to expressly specify a time and a place for delivery of the goods.
b.
Unless both the seller and the buyer are merchants, neither party is obligated to perform the contract in good faith.
c.
The seller may be excused from performance if the goods are accidentally destroyed before the risk of loss passes to the buyer.
d.
If the price of the goods is less than $500, the goods need not be identified to the contract for title to pass to the buyer.

A

Choice “c” is correct. Under Article 2 of the UCC the seller of goods may be excused from performance if the goods (subject matter of the contract) are accidentally destroyed before the risk of loss passes to the buyer and performance is thereby rendered impossible.
Choice “b” is incorrect since the UCC requires that all parties to a contract act in good faith.
Choice “a” is incorrect. In the absence of a place for delivery the UCC provides for the “seller’s place” of business. In the absence of a time for delivery the UCC provides a reasonable time.
Choice “d” is incorrect. In all cases the goods must be identified to the contract for title to pass to the buyer.

57
Q

To which of the following transactions does the common law Statute of Frauds not apply?
a.
Contracts for the sale of real estate.
b.
Agreements made in consideration of marriage.
c.
Contracts that can be performed within one year.
d.
Promises to pay the debt of another.

A

Choice “c” is correct. Contracts which by their terms cannot be performed within a year are within the common law Statute of Frauds. The fact that a contract may be performed within a year does not bring the contract within the Statute of Frauds. A contract to perform weekly landscaping services for the next three years must be in writing. A contract to plant three trees within the next two years is not within the statute of frauds.
Choice “a” is incorrect. All contracts for the sale of real property are within the common law Statute of Frauds and so must be evidenced by a writing to be enforceable.
Choice “b” is incorrect. Contracts for which the consideration is marriage are within the common law Statute of Frauds and so must be evidenced by a writing to be enforceable.
Choice “d” is incorrect. Promises to pay the debt of another are suretyship promises and are within the common law Statute of Frauds. Therefore, they must be evidenced by a writing to be enforceable.

58
Q

Which of the following types of conduct renders a contract void?
a.
Duress through improper threats.
b.
Duress through physical compulsion.
c.
Undue influence by a dominant party in a confidential relationship.
d.
Mutual mistake as to facts forming the basis of the contract.

A

Choice “b” is correct. Duress through physical harm or the threat of physical harm renders a contract void rather than merely voidable.
Choice “d” is incorrect. Mutual mistake renders a contract voidable by either party, rather than void.
Choice “c” is incorrect. Use of undue influence renders a contract voidable at the option of the party influenced.
Choice “a” is incorrect. Duress through improper threats is too broad to be correct. If the improper threats regard social or economic disadvantage, the contract is merely voidable. If the threats involve physical violence, then the contract is void.

59
Q

On day 1, Jackson, a merchant, mailed Sands a signed letter that contained an offer to sell Sands 500 electric fans at $10 per fan. The letter was received by Sands on day 3. The letter contained a promise not to revoke the offer but no expiration date. On day 4, Jackson mailed Sands a revocation of the offer to sell the fans. Sands received the revocation on day 6. On day 7, Sands mailed Jackson an acceptance of the offer. Jackson received the acceptance on day 9. Under the Sales Article of the UCC, was a contract formed?
a.
No contract was formed because Sands received the revocation of the offer before Sands accepted the offer.
b.
No contract was formed because the offer failed to state an expiration date.
c.
A contract was formed on the day Sands mailed the acceptance to Jackson.
d.
A contract was formed on the day Jackson received Sands’ acceptance.

A

Choice “c” is correct. Formation of a contract requires acceptance of an offer before the offer is terminated. Jackson sent Sands an offer and then sent a revocation of the offer, and Sands accepted after receiving the revocation. However, the revocation was invalid because the offer was a merchant’s firm offer―an offer made by a merchant in writing giving assurances that it will be kept open. Such offers are irrevocable for the time stated, or if no time is stated for a reasonable time, but in no event longer than three months. Thus, Jackson’s offer was irrevocable for a reasonable time. It was accepted within a reasonable time (a few days after the offer was received). Moreover, the contract was accepted when Sands mailed the acceptance because of the mailbox rule―an acceptance is effective upon being dispatched.

60
Q
What type of conduct generally will make a contract voidable?
	a.	
Contracting with a person under guardianship.
	b.	
Fraud in the execution.
	c.	
Fraud in the inducement.
	d.	
Physical coercion.
A

Choice “c” is correct. If a person is defrauded into entering into a contract because its terms or the surrounding circumstances are not as represented (that is, fraud in the inducement), the contract is merely voidable.
Choice “b” is incorrect. Fraud in the execution (that is, the party did not know that he was signing a contract) renders a contract void.
Choice “d” is incorrect. Physical coercion (or the threat of physical coercion) is a type of duress that renders a contract void rather than voidable.
Choice “a” is incorrect. Entering into a contract with a person who is without capacity and has a guardian appointed (that is, after a court has declared the person incompetent) renders the contract void.

61
Q

Under the Sales Article of the UCC, which of the following statements is correct regarding risk of loss and title to the goods under a sale or return contract?
a.
Title and risk of loss rest with the buyer until the goods are returned to the seller.
b.
Title remains with the seller until the buyer approves or accepts the goods, but risk of loss passes to the buyer immediately following delivery of the goods to the buyer.
c.
Title and risk of loss remain with the seller until the buyer pays for the goods.
d.
Title and risk of loss are shared equally between the buyer and the seller.

A

Choice “a” is correct. In a sale or return, the buyer has title and risk of loss unless and until the goods are returned to the seller.

62
Q

An appliance seller promised a restaurant owner that a home dishwasher would fulfill the dishwashing requirements of a large restaurant. The dishwasher was purchased but it was not powerful enough for the restaurant. Under the Sales Article of the UCC, what warranty was violated?
a.
The implied warranty of merchantability.
b.
The express warranty against infringement.
c.
The implied warranty of marketability.
d.
The express warranty that the goods conform to the seller’s promise.

A

Choice “d” is correct. Any affirmation of fact or promise that becomes part of the basis of the bargain creates an express warranty.
Choice “c” is incorrect. There is no warranty under the Sales Article entitled the “warranty of marketability.”
Choice “a” is incorrect. The warranty of merchantability is a promise that the goods will be fit for their ordinary purposes. Nothing here indicates that the home dishwasher would be unfit for its ordinary purpose of washing dishes in a home.
Choice “b” is incorrect. The warranty against infringement is a promise that the goods sold do not infringe on another’s patents or copyrights. Nothing indicates that this warranty was breached here.

63
Q
Under the Sales Article of the UCC, which of the following circumstances will relieve a buyer from the obligation of accepting a tender or delivery of goods?
I.
If the goods do not meet the buyer's needs at the time of the tender or delivery.
II.
If the goods at the time of the tender or delivery do not exactly conform to the requirements of the contract.
	a.	
Both I and II.
	b.	
Neither I nor II.
	c.	
I only.
	d.	
II only.
A

Choice “d” is correct. A buyer may reject goods if they do not conform to the contract in any way. This is known as the perfect tender doctrine. However, the mere fact that the goods do not meet the buyer’s needs at the time of tender or delivery is not a ground for rejection if the goods conform to the contract.

64
Q

Under the Sales Article of the UCC, which of the following statements is correct regarding a good faith requirement that must be met by a merchant?
a.
The merchant must charge the lowest available price for the product in the geographic market.
b.
The merchant must adhere to all written and oral terms of the sales contract.
c.
The merchant must observe the reasonable commercial standards of fair dealing in the trade.
d.
The merchant must provide more extensive warranties than the minimum required by law.

A

Choice “c” is correct. The UCC imposes an obligation of good faith on both parties to a contract. For merchants, this includes the duty to observe reasonable commercial standards.
Choice “b” is incorrect. Under the parol evidence rule, a merchant would not have to adhere to oral statements made before a written contract was made if the written contract appears to be a total integration of the entire deal.
Choice “d” is incorrect. A merchant need not go beyond the warranties required by law.
Choice “a” is incorrect. There is no rule requiring merchants to sell at the lowest prices.

65
Q

Under the Sales Article of the UCC, which of the following statements is correct regarding a seller’s obligation under a F.O.B. destination contract?
a.
The seller is required to tender delivery of conforming goods to a carrier who delivers to a destination specified by the buyer.
b.
The seller is required to tender delivery of conforming goods at the buyer’s place of business.
c.
The seller is required to arrange for the buyer to pick up the conforming goods at a specified destination.
d.
The seller is required to tender delivery of conforming goods at a specified destination.

A

Choice “d” is correct. Under an F.O.B. destination contract, the seller has the risk of loss until he places conforming goods into the buyer’s hands at the named destination, not necessarily the buyer’s place of business.

66
Q
Under the Sales Article of the UCC, in an auction announced in explicit terms to be without reserve, when may an auctioneer withdraw the goods put up for sale?
I.
At any time until the auctioneer announces completion of the sale.
II.
If no bid is made within a reasonable time.
	a.	
Neither I nor II.
	b.	
I only.
	c.	
Either I or II.
	d.	
II only.
A

Choice “d” is correct. In an auction without reserve, the goods must be sold if an offer is made. Of course, if no offer is made within a reasonable time, the goods need not be sold. Item I describes a sale with reserve.

67
Q

Patch, a frequent shopper at Soon-Shop Stores, received a rain check for an advertised sale item after Soon-Shop’s supply of the product ran out. The rain check was in writing and stated that the item would be offered to the customer at the advertised sale price for an unspecified period of time. A Soon-Shop employee signed the rain check. When Patch returned to the store one month later to purchase the item, the store refused to honor the rain check. Under the Sales Article of the UCC, will Patch win a suit to enforce the rain check?
a.
No, because one month is too long a period of time for a rain check to be effective.
b.
Yes, because Soon-Shop is required to have sufficient supplies of the sale item to satisfy all customers.
c.
Yes, because the rain check met the requirements of a merchant’s firm offer even though no effective time period was stated.
d.
No, because the rain check did not state the effective time period necessary to keep the offer open.

A

Choice “c” is correct. The rain check satisfied the requirements of a “firm offer” because it involved the sale of goods, the seller was a merchant, it was in writing and signed by the merchant, and the writing included words of firmness (i.e., a promise to keep the offer open). If no time is stated, as in this case, the offer is irrevocable for a reasonable time, up to three months.
Choice “a” is incorrect. A merchant’s firm offer can be irrevocable for up to three months.
Choice “d” is incorrect. If a merchant’s firm offer does not indicate how long the offer will be kept open, it is irrevocable for a reasonable time, not to exceed three months.
Choice “b” is incorrect. There is no such rule under the UCC Sales Article, and, as a practical matter, it would be difficult to predict the supply needed.

68
Q

A sheep rancher agreed, in writing, to sell all the wool shorn during the shearing season to a weaver. The contract failed to establish the price and a minimum quantity of wool. After the shearing season, the rancher refused to deliver the wool. The weaver sued the rancher for breach of contract. Under the Sales Article of the UCC, will the weaver win?
a.
No, because quantity cannot be omitted for a contract to be enforceable.
b.
No, because the omission of price and quantity terms prevents the formation of a contract.
c.
Yes, because this was an output contract.
d.
Yes, because both price and quantity terms were omitted.

A

Choice “c” is correct. Under the UCC, a contract to buy all of one’s requirements or to sell all of one’s output is valid even though an exact quantity is not stated. In addition, price and time for delivery are not essential terms under the UCC. As a general rule, the only essential term under the UCC is quantity, and an output or requirements term is considered a sufficiently precise quantity.
Choice “d” is incorrect. First, the contract has a quantity term, the rancher’s output for the season. Thus, this choice is factually incorrect. Second, if it were missing a quantity term, that would be a reason preventing enforcement.
Choice “a” is incorrect. Although it is true that an agreement is unenforceable under the Sales Article if it lacks a quantity term, the UCC treats output as an acceptable quantity term.
Choice “b” is incorrect because when a price term is omitted, the UCC implies a reasonable price, and the contract here does have a quantity term because output is considered an acceptable quantity.

69
Q

EG Door Co., a manufacturer of custom exterior doors, verbally contracted with Art Contractors to design and build a $2,000 custom door for a house that Art was restoring. After EG had completed substantial work on the door, Art advised EG that the house had been destroyed by fire and Art was canceling the contract. EG finished the door and shipped it to Art. Art refused to accept delivery. Art contends that the contract cannot be enforced because it violated the Statute of Frauds by not being in writing. Under the Sales Article of the UCC, is Art’s contention correct?
a.
Yes, because the contract cannot be fully performed due to the fire.
b.
No, because the cancellation of the contract was not made in writing.
c.
No, because the goods were specially manufactured for Art and cannot be resold in EG’s regular course of business.
d.
Yes, because the contract was not in writing.

A
Choice "c" is correct. The Statute of Frauds requires contracts involving the sales of goods to be in writing if they exceed $500 (MYLEGS). However, if any of these exceptions apply, an oral contract will be enforceable:
Specially manufactured (custom) goods
Written confirmation between merchants
Admission in court
Performance
70
Q

Under the Sales Article of the UCC, when a contract for the sale of goods stipulates that the seller ship the goods by common carrier “F.O.B. purchaser’s loading dock,” which of the parties bears the risk of loss during shipment?
a.
The seller, because risk of loss remains with the seller until the goods are accepted by the purchaser.
b.
The purchaser, because risk of loss passes when the goods are delivered to the carrier.
c.
The seller, because risk of loss passes only when the goods reach the purchaser’s loading dock.
d.
The purchaser, because title to the goods passes at the time of shipment.

A

Choice “c” is correct. When a contract for the sale of goods includes an F.O.B. (free on board) delivery term, that term controls risk of loss. The seller has the risk of loss until the goods are delivered at the location named after the F.O.B. term. Here, the location is the purchaser’s loading dock, so risk of loss remains with the seller until the goods are delivered there.
Choice “b” is incorrect. When the delivery term is F.O.B. purchaser’s loading dock, risk of loss does not pass to the purchaser until the goods are delivered at the purchaser’s loading dock.
Choice “d” is incorrect. Under the UCC, risk of loss does not depend upon title, but rather upon the delivery term.
Choice “a” is incorrect. Risk of loss under an F.O.B. term passes when the goods are delivered at the named location, not when the goods are accepted by the purchaser.

71
Q

Under the Sales Article of the UCC, a firm offer will be created only if the:
a.
Offeree is a merchant.
b.
Offer is made by a merchant in a signed writing.
c.
Offeree gives some form of consideration.
d.
Offer states the time period during which it will remain open

A

Choice “b” is correct. A firm offer (an offer that must remain open despite the absence of consideration) can be made only by merchants and must be in a signed writing.
Choice “d” is incorrect. If a firm offer does not state its period of irrevocability, it will remain open for a reasonable time not to exceed three months.
Choice “c” is incorrect. A firm offer is an offer that must remain open despite the lack of consideration.
Choice “a” is incorrect. The offeree need not be a merchant in a firm offer situation; only the offeror need be a merchant.

72
Q

Under the Sales Article of the UCC, the warranty of title:
a.
Applies only if the seller is a merchant.
b.
Applies only if it is in writing and signed by the seller.
c.
Provides that the seller deliver the goods free from any lien of which the buyer lacked knowledge when the contract was made.
d.
Provides that the seller cannot disclaim the warranty if the sale is made to a bona fide purchaser for value.

A

Choice “c” is correct. The warranty of title is a guarantee from the seller that the goods are delivered free of all liens of which the buyer is unaware.
Choice “d” is incorrect. The warranty of title can be disclaimed by specific language or circumstance.
Choice “b” is incorrect. The warranty of title arises automatically in every sale of goods; it need not be in writing.
Choice “a” is incorrect. The warranty of title arises automatically in every sale of goods; even when the seller is not a merchant.

73
Q

An appliance seller promised a restaurant owner that a home dishwasher would fulfill the dishwashing requirements of a large restaurant. The dishwasher was purchased but it was not powerful enough for the restaurant. Under the Sales Article of the UCC, what warranty was violated?
a.
The implied warranty of merchantability.
b.
The express warranty against infringement.
c.
The express warranty that the goods conform to the seller’s promise.
d.
The implied warranty of marketability.

A

Choice “c” is correct. Any affirmation of fact or promise that becomes part of the basis of the bargain creates an express warranty.
Choice “d” is incorrect. There is no warranty under the Sales Article entitled the “warranty of marketability.”
Choice “a” is incorrect. The warranty of merchantability is a promise that the goods will be fit for their ordinary purposes. Nothing here indicates that the home dishwasher would be unfit for its ordinary purpose of washing dishes in a home.
Choice “b” is incorrect. The warranty against infringement is a promise that the goods sold do not infringe on another’s patents or copyrights. Nothing indicates that this warranty was breached here.

74
Q
To establish a cause of action based on strict liability in tort for personal injuries that result from the use of a defective product, one of the elements the injured party must prove is that the seller:
	a.	
Sold the product to the injured party.
	b.	
Sold the product in a defective condition.
	c.	
Failed to exercise due care.
	d.	
Was aware of the defect in the product.
A

Choice “b” is correct. An action for strict product liability will succeed only if the product was in a defective condition when sold, the seller was in the business of selling goods, the defect caused the plaintiff’s injury, and the product was expected to and did reach the consumer without substantial change.

75
Q
Under the Sales Article of the UCC, which of the following factors is most important in determining who bears the risk of loss in a sale of goods contract?
	a.	
Title to the goods.
	b.	
The contract's shipping terms.
	c.	
How the goods were lost.
	d.	
The method of shipping the goods.
A

Choice “b” is correct. Assuming that the loss occurred under a shipment contract, risk of loss under the UCC is controlled by the shipping terms, not by title.
Choice “d” is incorrect. The method of shipping the goods (e.g., car vs. train) is irrelevant to who bears the risk of loss under the UCC; the shipping terms are the key when the contract is a shipment contract.
Choice “a” is incorrect. Risk of loss under the UCC is controlled by the shipping terms in a shipment contract, not by title.
Choice “c” is incorrect. Risk of loss under the UCC is controlled by the shipping terms in a shipment contract, not by how the goods were lost.

76
Q
Under the Sales Article of the UCC, in an F.O.B. place of shipment contract, the risk of loss passes to the buyer when the goods:
	a.	
Are delivered to the carrier.
	b.	
Reach the buyer's loading dock.
	c.	
Are placed on the seller's loading dock.
	d.	
Are identified to the contract.
A

Choice “a” is correct. In an F.O.B. place of shipment contract, risk of loss passes when the goods are placed in the hands of a carrier at the seller’s loading dock.
Choice “d” is incorrect. Risk of loss would pass on identification only if the parties specifically so provided.
Choice “c” is incorrect. In an F.O.B. place of shipment contract, it is not sufficient just to get the goods to the loading dock; risk does not pass until the goods are placed in the hands of a carrier there.
Choice “b” is incorrect. In an F.O.B. place of shipment contract, risk of loss passes when the goods are placed in the hands of a carrier at the seller’s loading dock.

77
Q

Under the Sales Article of the UCC, and unless otherwise agreed to, the seller’s obligation to the buyer is to:
a.
Set aside conforming goods for inspection by the buyer before delivery.
b.
Deliver all goods called for in the contract to a common carrier.
c.
Hold conforming goods and give the buyer whatever notification is reasonably necessary to enable the buyer to take delivery.
d.
Deliver the goods to the buyer’s place of business.

A

Choice “c” is correct. Absent an agreement otherwise, the seller is not obligated to deliver the conforming goods to the buyer, but merely needs to hold them for the buyer’s disposition.
Choice “d” is incorrect. Absent an agreement otherwise, a seller has no duty to deliver the conforming goods to the buyer.
Choice “b” is incorrect. Absent an agreement otherwise, a seller has no duty to deliver the conforming goods to a common carrier.
Choice “a” is incorrect. Absent an agreement otherwise, a seller need not hold the conforming goods aside for inspection before delivery.

78
Q
Under the Sales Article of the UCC, which of the following statements regarding liquidated damages is (are) correct?
I.
The injured party may collect any amount of liquidated damages provided for in the contract.
II.
The seller may retain a deposit of up to $500 when a buyer defaults even if there is no liquidated damages provision in the contract.
	a.	
I only.
	b.	
Neither I nor II.
	c.	
Both I and II.
	d.	
II only.
A

Choice “d” is correct. I: An injured party cannot necessarily collect “any amount” of liquidated damages specified in a contract. UCC 2-718(1) restricts recovery to reasonable liquidated damages; any amounts above a reasonable amount are considered unenforceable penalties. II: Under UCC 2-718(2)(b), a seller can usually retain up to $500 of the buyer’s deposit on the buyer’s breach.

79
Q

Under the Sales Article of the UCC, which of following statements is correct?
a.
The contract must involve the sale of goods for a price of more than $500.
b.
The obligations of the parties to the contract must be performed in good faith.
c.
Merchants and nonmerchants are treated alike.
d.
None of the provisions of the UCC may be disclaimed by agreement.

A

Choice “b” is correct. The Sales Article imposes a duty of good faith on all parties.
Choice “c” is incorrect. Certain provisions of the Sales Article differentiate between merchants and nonmerchants (e.g., only a merchant makes the implied warranty of merchantability).
Choice “a” is incorrect. The Sales Article applies to all sales of goods. The $500 limit refers to the Statute of Frauds within the Sales Article.
Choice “d” is incorrect. Most provisions of the Sales Article can be varied by agreement (e.g., warranties may be disclaimed).

80
Q

Under the Sales Article of the UCC, which of the following statements is correct regarding the warranty of merchantability arising when there has been a sale of goods by a merchant seller?
a.
The warranty arises when the buyer relies on the seller’s skill in selecting the goods purchased.
b.
The warranty must be in writing.
c.
The warranty arises as a matter of law when the seller ordinarily sells the goods purchased.
d.
The warranty cannot be disclaimed.

A

Choice “c” is correct. The warranty of merchantability is implied whenever a merchant (one who ordinarily sells goods of the kind sold) sells goods. UCC 2-314
Choice “b” is incorrect. The warranty of merchantability is an implied warranty—it need not be in writing.
Choice “a” is incorrect. The warranty of merchantability is implied whenever a merchant (one who ordinarily sells goods of the kind sold) sells goods. The warranty that can arise from reliance is the warranty of fitness for particular purpose.
Choice “d” is incorrect. The warranty of merchantability can be specifically disclaimed or by words such as “as is.”

81
Q

High sues the manufacturer, wholesaler, and retailer for bodily injuries caused by a power saw High purchased. Which of the following statements is correct under strict liability theory?
a.
High may recover even if he cannot show any negligence was involved.
b.
The manufacturer will avoid liability if it can show it followed the custom of the industry.
c.
Contributory negligence on High’s part will always be a bar to recovery.
d.
Privity will be a bar to recovery insofar as the wholesaler is concerned if the wholesaler did not have a reasonable opportunity to inspect.

A

Choice “a” is correct. An action for strict product liability does not require a showing of negligence. The product must have been unreasonably dangerous when it left the seller’s hands.
Choice “c” is incorrect. In an action for strict liability in tort, High’s contributory negligence would be irrelevant.
Choice “b” is incorrect. Custom in the industry is good evidence that there was no negligence, but a whole industry can be found to be following a negligent practice, and industry custom is not a defense in a strict liability product liability case.
Choice “d” is incorrect. The fact that the wholesaler did not have an opportunity to inspect is not a defense.

82
Q

Under the Sales Article of the UCC, which of the following events will release the buyer from all its obligations under a sales contract?
a.
Impracticability of delivery under the terms of the contract.
b.
Anticipatory repudiation by the buyer that is retracted before the seller cancels the contract.
c.
Destruction of the goods after risk of loss passed to the buyer.
d.
Refusal of the seller to give written assurance of performance when reasonably demanded by the buyer.

A

Choice “d” is correct. Failure to give adequate assurances when reasonably demanded is a form of anticipatory repudiation. It constitutes a breach and discharges the buyer.
Choice “c” is incorrect. If the goods are destroyed after the risk of loss passes to the buyer, the buyer is obligated to pay for the goods since the buyer had the risk.
Choice “a” is incorrect. If the delivery terms are impracticable, the buyer must accept delivery by some other reasonable means.
Choice “b” is incorrect. If the buyer repudiates, the seller has the right to hold the buyer to the contract; the buyer is not discharged unless the seller cancels. If the buyer retracts the repudiation in a timely fashion, the contract is treated as if there were no repudiation and the buyer remains liable in full.

83
Q

Rowe Corp. purchased goods from Stair Co. that were shipped C.O.D. Under the Sales Article of the UCC, which of the following rights does Rowe have?
a.
The right to possession of the goods before paying.
b.
The right to delay payment for a reasonable period of time.
c.
The right to inspect the goods before paying.
d.
The right to reject nonconforming goods.

A

Choice “d” is correct. Under a C.O.D. contract, a buyer has a reasonable time after delivery in which to inspect the goods and reject them if they are nonconforming.
Choice “c” is incorrect. In a C.O.D. contract, there is no right to inspect the goods before delivery, as there is a duty to pay cash on delivery.
Choice “a” is incorrect. Under the Sales Article the general rule is that payment and possession are concurrent conditions; there is no general right to possession before payment.
Choice “b” is incorrect. A C.O.D. contract requires payment on delivery.

84
Q

Under the UCC Sales Article, which of the following statements is correct concerning a contract involving a merchant seller and a non-merchant buyer?
a.
The contract will be either a sale or return or sale on approval contract.
b.
The contract may not involve the sale of personal property with a price of more than $500.
c.
Whether the UCC Sales Article is applicable does not depend on the price of the goods involved.
d.
Only the seller is obligated to perform the contract in good faith.

A

Choice “c” is correct. The Sales Article applies to all contracts for the sale of goods, regardless of price.
Choice “d” is incorrect. All parties are bound by the obligation of good faith under the UCC.
Choice “a” is incorrect. The presumption is that all sales are final. A sale or return or sale on approval (both of which allow the return of the goods) is available only if the parties so provide.
Choice “b” is incorrect. The Sales Article covers all sales of goods. If the purchase price is $500 or more, a writing may be required to enforce the contract under the Sales Article’s Statute of Frauds, but the Sales Article still applies.

85
Q

Vick bought a used boat from Ocean Marina that disclaimed “any and all warranties” in connection with the sale. Ocean was unaware the boat had been stolen from Kidd. Vick surrendered it to Kidd when confronted with proof of the theft. Vick sued Ocean. Who is likely to prevail and why?
a.
Ocean, because of the disclaimer of warranties.
b.
Vick, because the implied warranty of title has been breached.
c.
Ocean, because Vick surrendered the boat to Kidd.
d.
Vick, because a merchant cannot disclaim implied warranties.

A

Choice “b” is correct. Every sale includes a warranty that the seller has title unless the warranty is specifically disclaimed or the facts give the buyer notice that there is no such warranty. The warranty is not disclaimed by general disclaimers such as the one here disclaiming “any and all warranties.”

86
Q

Quick Corp. agreed to purchase 200 typewriters from Union Suppliers, Inc. Union is a wholesaler of appliances and Quick is an appliance retailer. The contract required Union to ship the typewriters to Quick by common carrier, “FOB Union Suppliers, Inc. Loading Dock.” Which of the parties bears the risk of loss during shipment?
a.
Quick, because title to the typewriters passed to Quick at the time of shipment.
b.
Union, because both parties are merchants.
c.
Quick, because the risk of loss passes when the typewriters are delivered to the carrier.
d.
Union, because the risk of loss passes only when Quick receives the typewriters.

A

Choice “c” is correct. In an FOB contract, the seller has the risk of loss up to the named location. Here, the location named is the seller’s own loading dock. Thus, the risk of loss passes as soon as the goods are off the loading dock and in the hands of the carrier. UCC 2-319
Choice “d” is incorrect. In an FOB contract, risk of loss remains with the seller only up to the named location, not until the goods are received by the seller.
Choice “b” is incorrect. In a shipment contract, the time that risk of loss passes is dependent on the terms of the contract and not on whether the parties are merchants. Even where shipment is not contemplated, risk of loss depends on whether the seller is a merchant. The status of the buyer is irrelevant.
Choice “a” is incorrect. Although it is true that title passed to the buyer on delivery to the carrier, risk of loss does not necessarily follow title.

87
Q

Webstar Corp. orally agreed to sell Northco Inc. a computer for $20,000. Northco sent a signed purchase order to Webstar confirming the agreement. Webstar received the purchase order and did not respond. Webstar refused to deliver the computer to Northco, claiming that the purchase order did not satisfy the UCC Statute of Frauds because it was not signed by Webstar. Northco sells computers to the general public and Webstar is a computer wholesaler. Under the UCC Sales Article, Webstar’s position is:
a.
Correct because it was the party against whom enforcement of the contract is being sought.
b.
Correct because the purchase price of the computer exceeded $500.
c.
Incorrect because only the buyer in a sale-of-goods transaction must sign the contract.
d.
Incorrect because it failed to object to Northco’s purchaser order.

A

Choice “d” is correct. Although the contract here requires a writing under the Statute of Frauds signed by the party to be charged, the Sales Article provides a “confirmatory memo” or “written confirmation” exception between merchants. If one merchant sends the other a memo of their contract sufficient to bind the sender, it will bind the recipient as well unless the recipient objects within a reasonable time. UCC 2-201

88
Q
Under the UCC Sales Article, which of the following legal remedies would a buyer not have when a seller fails to transfer and deliver goods identified to the contract?
	a.	
Recover the identified goods (capture).
	b.	
Suit for punitive damages.
	c.	
Purchase substitute goods (cover).
	d.	
Suit for specific performance.
A

Choice “b” is correct. The Sales Article does not provide for punitive damages.
Choice “d” is incorrect. The Sales Article allows an aggrieved buyer specific performance when substitute goods cannot be purchased; e.g., where the subject matter is unique.
Choice “c” is incorrect. “Cover” (purchase of substitute goods) is one of the basic remedies available to the buyer where a seller refuses to deliver goods.
Choice “a” is incorrect. Where goods have been identified (and the buyer has paid part of the purchase price), the buyer may take possession of the goods through replevin.

89
Q

Which of the following statements applies to a sale on approval under the UCC Sales Article?
a.
Both the buyer and seller must be merchants.
b.
Risk of loss for the goods passes to the buyer when the goods are accepted after the trial period.
c.
The buyer must be purchasing the goods for resale.
d.
Title to the goods passes to the buyer on delivery of the goods to the buyer.

A

Choice “b” is correct. In a sale on approval, risk of loss passes to the buyer on approval of the goods.
Choice “a” is incorrect. A sale on approval does not require that either party be a merchant.
Choice “c” is incorrect. If the buyer is purchasing for resale, it generally is a sale or return. In any case, there is no such requirement for a sale on approval.
Choice “d” is incorrect. In a sale on approval, title does not pass to the buyer on delivery; rather it passes when the buyer approves the goods.

90
Q

Which of the following statements would not apply to a written contract governed by the provisions of the UCC Sales Article?
a.
The contract must involve the sale of goods for a price of $500 or more.
b.
The obligations of a nonmerchant may be different from those of a merchant.
c.
The obligations of the parties must be performed in good faith.
d.
The contract may involve the sale of personal property.

A

Choice “a” is correct. Any contract can be in writing under Article 2. If the contract is for the sale of goods for $500 or more a writing is required under the Statute of Frauds, but this does not mean that a writing is not permitted for contracts involving less money.
Choice “d” is incorrect. A written contract under Article 2 can involve personal property; it may not involve real property.
Choice “b” is incorrect. Some obligations under Article 2 do differ depending on whether merchants are involved (e.g., the implied warranty of merchantability arises only in sales by merchants).
Choice “c” is incorrect. All contracts under the UCC are subject to the good faith requirement.

91
Q

On May 2, Handy Hardware sent Ram Industries a signed purchase order that stated, in part, as follows:
“Ship for May 8 delivery 300 Model A-X socket sets at current dealer price. Terms 2/10/net 30.”
Ram received Handy’s purchase order on May 4. On May 5, Ram discovered that it had only 200 Model A-X socket sets and 100 Model W-Z socket sets in stock. Ram shipped the Model A-X and Model W-Z sets to Handy without any explanation concerning the shipment. The socket sets were received by Handy on May 8.
Which of the following statements concerning the shipment is correct?
a.
Ram’s shipment is a counteroffer.
b.
Ram’s shipment is an acceptance of Handy’s offer.
c.
Handy’s order can only be accepted by Ram shipping conforming goods.
d.
Handy’s order must be accepted by Ram in writing before Ram ships the socket sets.

A

Choice “b” is correct. Under Article 2 an offer can be accepted by a promise to ship or by prompt shipment. Shipment of nonconforming goods constitutes both an acceptance and a breach unless a notice is sent prior to shipping that the goods are only an accommodation.
Choice “a” is incorrect. Shipment of nonconforming goods is not a counteroffer unless accompanied by a notice of accommodation. Here, Handy shipped the socket sets without any explanation concerning the shipment.
Choice “d” is incorrect. In a goods contract, acceptance of an offer can be made merely by shipping goods.
Choice “c” is incorrect. Shipment of goods is an acceptance, whether or not the goods conform. If they do not conform, the shipment is also a breach.

92
Q

To establish a cause of action based on strict liability in tort for personal injuries resulting from using a defective product, one of the elements the plaintiff must prove is that the seller (defendant):
a.
Defectively designed the product.
b.
Was engaged in the business of selling the product.
c.
Was in privity of contract with the plaintiff.
d.
Failed to exercise due care.

A

Choice “b” is correct. Strict product liability applies only to persons in the business of selling the product involved, including manufacturers, wholesalers, and retailers.
Choice “d” is incorrect. Strict liability does not require a showing of fault. Thus, negligence need not be proved.
Choice “c” is incorrect. Privity is not required in tort. Thus, an action for strict liability can be brought against anyone in the chain of distribution of the defective product, as long as the product was defective when it left that person’s hands.
Choice “a” is incorrect. Strict product liability is not limited to design defects; it is available for manufacturing defects as well. Even when there was a design defect, it is not necessary to show that the defendant is the party who designed the product.

93
Q

Bond purchased a painting from Wool, who is not in the business of selling art. Wool tendered delivery of the painting after receiving payment in full from Bond. Bond informed Wool that Bond would be unable to take possession of the painting until later that day. Thieves stole the painting before Bond returned. The risk of loss:
a.
Remained with Wool because Bond had not yet received the painting.
b.
Passed to Bond at the time the contract was formed and payment was made.
c.
Passed to Bond at Wool’s tender of delivery.
d.
Remained with Wool because the parties agreed on a later time of delivery.

A

Choice “c” is correct. Where the seller is not a merchant, risk of loss passes to the buyer upon tender of delivery of the goods.

94
Q

Smith contracted in writing to sell Peters a used personal computer for $600. The contract did not specifically address the time for payment, place of delivery, or Peters’ right to inspect the computer. Which of the following statements is correct?
a.
Peters may not pay for the computer using a personal check unless Smith agrees.
b.
Peters is entitled to inspect the computer before paying for it.
c.
Smith is not entitled to payment until 30 days after Peters receives the computer.
d.
Smith is obligated to deliver the computer to Peters’ home.

A

Choice “b” is correct. A purchaser has a right to inspect goods before paying for them unless the contract provides otherwise.
Choice “d” is incorrect. If the contract is silent, delivery is deemed to be due at the seller’s place of business, or if the seller has none, at the seller’s home. Thus, delivery here would be at Smith’s home.
Choice “a” is incorrect. A seller must accept a check as payment or give the buyer additional time to obtain cash.
Choice “c” is incorrect. Unless otherwise agreed, payment and delivery are simultaneous conditions. Thus, payment here would be due on delivery.

95
Q
Cara Fabricating Co. and Taso Corp. agreed orally that Taso would custom manufacture a compressor for Cara at a price of $120,000. After Taso completed the work at a cost of $90,000, Cara notified Taso that the compressor was no longer needed. Taso is holding the compressor and has requested payment from Cara. Taso has been unable to resell the compressor for any price. Taso incurred storage fees of $2,000. If Cara refuses to pay Taso and Taso sues Cara, the most Taso will be entitled to recover is:
	a.	
$120,000
	b.	
$105,000
	c.	
$92,000
	d.	
$122,000
A

Choice “d” is correct. Despite the fact that the contract here was oral and for the sale of goods for $500 or more, it is enforceable under the Statute of Frauds because the compressor constitutes specially manufactured goods, and so the contract falls within an exception to the Statute of Frauds. A seller can bring an action for the full contract price when it is unable to sell the goods that a buyer refuses to accept. The seller can also recover incidental damages such as costs of storage. Thus, Taso would be able to recover the full $120,000 contract price plus the $2,000 in storage fees.

96
Q

Grill deals in the repair and sale of new and used clocks. West brought a clock to Grill to be repaired. One of Grill’s clerks mistakenly sold West’s clock to Hone, another customer. Under the Sales Article of the UCC, will West win a suit against Hone for the return of the clock?
a.
Yes, because Grill could not convey good title to the clock.
b.
No, because Grill is a merchant to whom goods had been entrusted.
c.
Yes, because the clerk was negligent in selling the clock.
d.
No, because the clerk was not aware that the clock belonged to West.

A

Choice “b” is correct. Generally, a person cannot pass on better title than the person has. However, if the owner of goods entrusts them to a merchant who deals in goods of that kind, and the merchant sells them in the ordinary course of the merchant’s business, then the merchant has the power to transfer title to the goods. West was the owner of the clock and he entrusted the clock to Grill, who deals in new and used clocks. Grill sold the clock to Hone in the ordinary course of business. Thus, Grill had the power to pass on good title to the clock and did so when one of Grill’s clerks mistakenly sold the clock to Hone. Because Hone has title to the clock, West cannot recover it from Hone. However, West does have an action for damages against Grill.

97
Q
Thorn purchased a used entertainment system from Sound Corp. The sales contract stated that the entertainment system was being sold "as is." Under the Sales Article of the UCC, which of the following statements is (are) correct regarding the seller's warranty of title and against infringement?
I.
Including the term "as is" in the sales contract is adequate communication that the seller is conveying the entertainment system without warranty of title and against infringement.
II.
The seller's warranty of title and against infringement may be disclaimed at any time after the contract is formed.
	a.	
Neither I nor II.
	b.	
Both I and II.
	c.	
II only.
	d.	
I only.
A

Choice “a” is correct. Under the Sales Article, all sales of goods include a warranty that the seller has title to the goods being sold unless the warranty is specifically disclaimed or the circumstances of the sale indicate that no such warranty is being made (e.g., a sheriff’s sale). A disclaimer must be made before or contemporaneously with the sale. A later disclaimer would be ineffective. Thus, neither I nor II is correct.

98
Q

Under the Sales Article of the UCC, which of the following statements is correct regarding the creation of express warranties?
a.
Express warranties are not enforceable if made orally.
b.
Express warranties must contain formal words such as warranty or guarantee.
c.
Express warranties cannot be based on statements made in the seller’s promotional materials.
d.
Express warranties must be part of the basis of the bargain between buyer and seller.

A

Choice “d” is correct. To be an express warranty, the language must be part of the basis of the bargain.
Choice “b” is incorrect. An express warranty arises from any statement of fact or promise made by the seller, any description of the goods made by the seller, or any sample or model shown by the seller at a time when it could have become part of the basis of the bargain.
Choice “a” is incorrect. An express warranty may be made orally, in writing or by conduct (e.g., the showing of a model).
Choice “c” is incorrect. Express warranties can arise from any description of the goods given to the buyer before the contract is executed.

99
Q

Under the UCC Sales Article, the implied warranty of merchantability:
a.
Arises only in contracts involving a merchant seller and a merchant buyer.
b.
May be disclaimed by a seller’s oral statement that mentions merchantability.
c.
Is breached if the goods are not fit for all purposes for which the buyer intends to use the goods.
d.
Must be part of the basis of the bargain to be binding on the seller.

A

Choice “b” is correct. Under Article 2 of the UCC the implied warranty of merchantability may be disclaimed in a number of ways. One such way is by the use of an oral statement that mentions merchantability.
Choice “a” is incorrect. Under the UCC the implied warranty of merchantability can only be given where the seller is a “merchant” of goods. The buyer may be a merchant or a non-merchant.
Choice “c” is incorrect. This answer concerns the implied warranty of “fitness” and not merchantability. The warranty of merchantability mainly is a warranty that the goods are fit for ordinary purposes.
Choice “d” is incorrect. An “express” warranty must be part of the basis of the bargain; this is not true of the “implied” warranties.

100
Q

Yost Corp., a computer manufacturer, contracted to sell 15 computers to Ivor Corp., a computer retailer. The contract specified that delivery was to be made by truck to Ivor’s warehouse. Instead, Yost shipped the computers by rail. When Ivor claimed that Yost did not comply with the contract, Yost told Ivor that there had been a trucker’s strike when the goods were shipped. Ivor refused to pay for the computers. Under these circumstances, Ivor:
a.
Is obligated to pay for the computers because Yost made a valid substituted performance.
b.
Is obligated to pay for the computers because title to them passed to Ivor when Ivor received them.
c.
May return the computers and avoid paying for them because the contract was void under the theory of commercial impracticability.
d.
May return the computers and avoid paying for them because of the way Yost delivered them.

A

Choice “a” is correct. Under the UCC, if the parties agreed that delivery would be made in a certain way and the agreed method becomes impossible, the seller must arrange for other commercially reasonable transportation and the buyer must accept. Therefore, “d” is incorrect. Impracticability is not a defense, so “c” is incorrect too.
Choice “b” is incorrect. Ivor is obligated to pay for them because Yost was permitted to use a different means of transportation, when title passed is not relevant. Ivor claimed he did not have to pay because Yost did not use a truck for delivery.

101
Q
Gray Fabricating Co. and Pine Corp. agreed orally that Pine would custom manufacture a processor for Gray at a price of $80,000. After Pine completed the work at a cost of $60,000, Gray notified Pine that the processor was no longer needed. Pine is holding the processor and has requested payment from Gray. Pine has been unable to resell the processor for any price. Pine incurred storage fees of $1,000. If Gray refuses to pay Pine and Pine sues Gray, the most Pine will be entitled to recover is:
	a.	
$61,000
	b.	
$60,000
	c.	
$81,000
	d.	
$80,000
A

Choice “c” is correct. Where the buyer of specially manufactured goods breaches the contract and the seller is unable to resell the goods, the seller may recover as damages the contract price ($80,000) plus reasonable incidental expenses, such as the costs of storage ($1,000).
Note: An oral agreement in excess of $500 is binding in the sale of “special order goods” as an exception to the Statute of Frauds.

102
Q

On May 2, Lace Corp., an appliance wholesaler, offered to sell appliances worth $3,000 to Parco, Inc., a household appliance retailer. The offer was signed by Lace’s president, and provided that it would not be withdrawn before June 1. It also included the shipping terms: “F.O.B.-Parco’s warehouse.” On May 29, Parco mailed an acceptance of Lace’s offer. Lace received the acceptance on June 2.
If Lace inadvertently ships the wrong appliances to Parco and Parco rejects them two days after receipt, title to the goods will:
a.
Remain with Parco until the goods are returned to Lace.
b.
Pass to Parco when they are identified to the contract.
c.
Pass to Parco when they are shipped.
d.
Revert to Lace when they are rejected by Parco.

A

Choice “d” is correct. Once the buyer has rightfully rejected the non-conforming goods, the title to the goods reverts to the seller.
Choice “b” is incorrect. Title to goods passes when the parties agree, or if they do not agree, upon delivery. Title does not pass when the goods are identified to the contract.
Choice “c” is incorrect. Under the UCC, title generally passes when goods are delivered. Where the seller has the duty to deliver to a destination other than the seller’s place of business (here, the buyer’s warehouse), title does not pass until the goods are delivered at that location.
Choice “a” is incorrect. A rejection or other refusal by the buyer to receive or retain the goods, whether or not justified, or a justified revocation of acceptance revests title to the goods in the seller at the time of rejection or revocation of acceptance. Title does not remain with the buyer until the goals are returned to the seller.

103
Q

On May 2, Mason orally contracted with Acme Appliances to buy for $480 a washer and dryer for household use. Mason and the Acme salesperson agreed that delivery would be made on July 2. On May 5, Mason telephoned Acme and requested that the delivery date be moved to June 2. The Acme salesperson agreed with this request. On June 2, Acme failed to deliver the washer and dryer to Mason because of an inventory shortage. Acme advised Mason that it would deliver the appliances on July 2 as originally agreed. Mason believes that Acme has breached its agreement with Mason. Acme contends that its agreement to deliver on June 2 was not binding. Acme’s contention is:
a.
Incorrect, because the agreement to change the delivery date was binding.
b.
Correct, because Mason is not a merchant and was buying the appliances for household use.
c.
Incorrect, because Acme’s agreement to change the delivery date is a firm offer that cannot be withdrawn by Acme.
d.
Correct, because the agreement to change the delivery date was not in writing.

A

Choice “a” is correct. Under the UCC, a contract involving “goods” may be modified without the need for additional (new) consideration. Here, the subsequent modification is binding. Moreover, no writing was required for the modification because the contract price, as modified, was less than $500.

104
Q
Which of the following conditions must be met for an implied warranty of fitness for a particular purpose to arise in connection with a sale of goods?
I.
The warranty must be in writing.
II.
The seller must know that the buyer was relying on the seller in selecting the goods.
	a.	
Both I and II.
	b.	
II only.
	c.	
Neither I nor II.
	d.	
I only.
A

Choice “b” is correct. II only. The implied warranty of fitness is created when the seller of goods knows at the time of the sale that the buyer is relying on the seller to select goods fit for the purpose that the buyer told the seller about.
The implied warranty of fitness need not be in writing or even specifically mentioned.

105
Q

On February 15, Mazur Corp. contracted to sell 1,000 bushels of wheat to Good Bread, Inc. at $6.00 per bushel with delivery to be made on June 23. On June 1, Good advised Mazur that it would not accept or pay for the wheat. On June 2, Mazur sold the wheat to another customer at the market price of $5.00 per bushel. Mazur had advised Good that it intended to resell the wheat. Which of the following statements is correct?
a.
Mazur can resell the wheat only after June 23.
b.
Good can retract its anticipatory breach at any time before June 23.
c.
Mazur can successfully sue Good for the difference between the resale price and the contract price.
d.
Good can successfully sue Mazur for specific performance.

A

Choice “c” is correct. Good’s statement constituted an anticipatory repudiation (i.e., an unequivocal statement that the party would not perform). Anticipatory repudiation is an immediate breach, and it gives the nonbreaching party several options, including the option to treat the contract as being breached, reselling the goods, and recovering the difference between the contract price and the resale price.

106
Q

Cookie Co. offered to sell Distrib Markets 20,000 pounds of cookies at $1.00 per pound, subject to certain specified terms for delivery. Distrib replied in writing as follows:
“We accept your offer for 20,000 pounds of cookies at $1.00 per pound, weighing scale to have valid city certificate.”
Under the UCC:
a.
A contract will be formed only if Cookie agrees to the weighing scale requirement.
b.
No contract was formed because Distrib included the weighing scale requirement in its reply.
c.
A contract was formed between the parties.
d.
No contract was formed because Distrib’s reply was a counteroffer.

A

Choice “c” is correct. The UCC does not follow the mirror image rule; instead, generally anything that looks like an acceptance will operate as an acceptance, even if it contains new terms. In such a case, a contract generally is formed even if the offeror fails to agree to the new terms. Thus, even though the acceptance contained an additional term regarding the weighing scale, it is an effective acceptance. Indeed, even if this contract were at common law, the acceptance probably would have been found valid. The requirement of a valid city certificate for the weighing scale probably would be considered to be an implied condition of the offer.

107
Q
Party A contracts for the sale of widgets to Party B. Before the date on which performance is due, Party B notifies Party A that it will not perform. This is a(an):
	a.	
Anticipatory retribution.
	b.	
Perfect tender.
	c.	
Anticipatory repudiation.
	d.	
Rejection of performance.
A

Choice “c” is correct. Anticipatory repudiation occurs when either the buyer or the seller indicates in advance of performance that he or she will not perform.
Choice “a” is incorrect. There is no such legal term.
Choice “b” is incorrect. Perfect tender is a UCC doctrine that provides that a buyer of goods may reject unless the goods tendered conform completely with the contract.
Choice “d” is incorrect. Rejection of performance involves the refusal of a tender of performance by the other party.

108
Q
Cindy purchases a stereo system from Kookie at Kookie's yard sale. Kookie is an appliance salesman for J&J's Appliance store. Kookie offers to place the stereo system in Cindy's car, but Cindy asks Kookie to temporarily hold onto the stereo instead. Kookie agrees to keep the stereo system in his garage until Cindy can pick it up. A fire starts in the garage through no fault of Kookie and the stereo is destroyed. Who bears the burden of the loss?
I.
Cindy.
II.
Kookie.
	a.	
Both I and II.
	b.	
Neither I nor II.
	c.	
II only.
	d.	
I only.
A

Choice “d” is correct. If the seller is a nonmerchant, such as one selling personal goods at a yard sale, risk of loss passes to the buyer upon the seller’s tender of delivery of the goods to the buyer. Although Kookie would qualify as an appliance merchant, Kookie did not deal in goods of the kind sold here (a car stereo) and so was not a merchant with respect to the stereo. Moreover, Kookie tendered delivery when Kookie offered to place the stereo in Cindy’s car. Thus, the risk of loss passed to Cindy at that time.

109
Q
Generally, if parties to a contract for the sale of goods do not mention how the goods are to be delivered, and where no common carrier is to be used, Article 2 of the UCC requires that the delivery take place at:
I.
The seller's place of business.
II.
The buyer's place of business.
III.
Either the seller's or the buyer's place of business.
	a.	
I only.
	b.	
Neither I, II, nor III.
	c.	
II only.
	d.	
III only.
A

Choice “a” is correct. Generally, where there is no agreement in noncarrier cases, the buyer will usually be expected to pick up the goods at the seller’s place of business.

110
Q

Anson enters into a contract to buy computers from the Becker Computer Store. Before an interest in the computers can pass from Becker to Anson:
I.
Becker must verify the credit rating of Anson.
II.
Becker must obtain shipping information on the computers.
III.
The computers must exist and be identified as the goods designated in the contract.
a.
III only.
b.
I, II, and III.
c.
I only.
d.
II only.

A

Choice “a” is correct. Goods must be identified to the contract before the buyer can have any interest in the goods.

111
Q

To determine whether the products that a merchant sells to the public are merchantable, a key factor to examine is whether:
a.
The products are fit for the ordinary purpose for which such goods are used.
b.
The manufacturer violated any laws or regulations.
c.
The products are quality products that are fit for a specific purpose.
d.
The products are made according to the best manufacturing methods.

A

Choice “a” is correct. The implied warranty of merchantability is implied in every sale by a merchant seller. Among other things, it is a warranty that the goods sold are fit for the ordinary purpose for which they were designed. For instance, a chair is designed to be sat upon as its ordinary purpose. If it breaks when a person sits on it that would be a breach of the warranty of merchantability.

112
Q

Under the Sales Article of the UCC, which of the following requirements must be met for a writing to be an enforceable contract for the sale of goods?
a.
The writing must contain a term specifying the price of the goods.
b.
The writing must contain the signature of the party seeking to enforce the writing.
c.
The writing must contain a term specifying the quantity of the goods.
d.
The writing must contain the signatures of all parties to the writing.

A

Choice “c” is correct. Under the Sales Article, if parties’ contracts are incomplete, the Article has many gap filling provisions through which the contract may be completed. However, the courts will not enforce a contract that does not state the quantity of the goods bought and sold, either specifically or in terms of output of the seller or requirements of the buyer.

113
Q

When do title and risk of loss for conforming goods pass to the buyer under a shipment contract covered by the Sales Article of the UCC?
a.
When the goods arrive at their destination.
b.
When the goods are tendered to the buyer at their destination.
c.
When the goods are given to a common carrier.
d.
When the goods are identified and designated for shipment.

A

Choice “c” is correct. In a shipment contract, risk of loss and title pass to the buyer when the goods are placed in the hands of the carrier.
Choice “d” is incorrect. At identification, the buyer gains some rights in the goods (e.g., an insurable interest), but title and risk of loss do not pass at that time.
Choices “a” and “b” are incorrect. Risk of loss and title pass to the buyer when the goods reach their destination and are tendered only in a destination contract. The question here asks about a shipment contract, which is governed by a different rule.

114
Q

Under the Sales Article of the UCC, which of the following circumstances best describes how the implied warranty of fitness for a particular purpose arises in a sale of goods transaction?
a.
The seller knows the particular purpose for which the buyer will use the goods and knows the buyer is relying on the seller’s skill or judgment to select suitable goods.
b.
The seller knows the particular purpose for which the buyer will use the goods and the seller is a merchant in such goods.
c.
The buyer is purchasing the goods for a particular purpose and is relying on the seller’s skill or judgment to select suitable goods.
d.
The buyer is purchasing the goods for a particular purpose and the seller is a merchant in such goods.

A

Choice “a” is correct. The implied warranty of fitness for particular purpose arises when the seller knows the particular purpose for which the buyer will use the goods and that the buyer is relying on the seller to choose suitable goods.
Choice “c” is incorrect. This choice is not as good as choice “a” because it does not include the idea that the seller must know a particular purpose to which the buyer is to put the goods. It is not enough merely that the buyer has a particular purpose in mind. The buyer must inform the seller of the purpose for the warranty to arise.
Choice “d” is incorrect. This choice is incorrect not only for the same reason that choice “c” is incorrect, but also because the seller need not be a merchant for the warranty of fitness to arise; nonmerchant sellers can make the warranty.
Choice “b” is incorrect. The warranty does not arise unless both prongs are present; the buyer must inform the seller of a particular purpose and must rely on the seller to select suitable goods. The seller need not be a merchant.

115
Q
Which of the following is not eligible for copyright protection?
	a.	
A source code of a computer program.
	b.	
A process.
	c.	
A sculpture.
	d.	
The choreography in a movie.
A

Choice “b” is correct. Generally speaking, copyright law protects works in a tangible medium. Processes are not protected. All of the other choices are examples of works protected by copyright law.

116
Q
How long is a copyright obtained by an individual author valid?
	a.	
The life of the author plus 14 years.
	b.	
The life of the author plus 20 years.
	c.	
The life of the author plus 70 years.
	d.	
The life of the author plus 50 years.
A

Choice “c” is correct. A copyright owned by an individual is good for the individual’s life plus 70 years.
Choice “a” is incorrect. Some patents are good for only 14 years, but a copyright is good for life plus 70 years.
Choice “b” is incorrect. 20 years is the period that most patents are good for.
Choice “d” is incorrect. Neither copyright nor patents have 50 year measuring periods.

117
Q
How long is a patent for a new drug good for?
	a.	
Fourteen years.
	b.	
The inventor's life plus 20 years.
	c.	
The inventor's life plus 14 years.
	d.	
Twenty years.
A

Choice “d” is correct. Patents for machines, new drugs, and the like are good for 20 years. Unlike copyrights, the length of the inventor’s life does not factor into the validity period.

118
Q

Abbeycorp engineers were working on a way to solve a problem that arose on their assembly line. In doing so, they created a new robot. After seeing how well the robot performed, the president of Abbeycorp realized that other companies could use a similar robot on their assembly lines. Before marketing the machine to other companies, the president would like to obtain a patent. Which of the following is not an attribute of the machine that the engineers will have to show in order to obtain a patent?
a.
It is novel.
b.
It is not obvious to others who work in the field.
c.
It is in a tangible medium of expression.
d.
It is useful.

A

Choice “c” is correct. To obtain a patent, an applicant must show that the invention is novel, useful, and not obvious to a person who works in the field. There is no requirement that the invention be in a tangible medium. That is an attribute needed to obtain a copyright. Thus, choices “a”, “d”, and “b” are incorrect.

119
Q
In order for a patent to be issued for an invention, the applicant must show that the invention is:
	a.	
All of the above.
	b.	
Not obvious.
	c.	
Novel.
	d.	
Useful.
A

Choice “a” is correct. For a patent to issue, the applicant must show that the invention is novel, useful, and not obvious to those working in the field. Choice “a” is the only choice that reflects all three requirements.